CME4PAC

Online CME for Physician Assistants

Menu

Skip to content
  • Home
  • Membership Benefits
  • My Account
  • About Us
  • Log In
  • Contact Us
  • Ortho on the Go

Pediatric Orthopedics

Time limit: 0

Quiz-summary

0 of 48 questions completed

Questions:

  1. 1
  2. 2
  3. 3
  4. 4
  5. 5
  6. 6
  7. 7
  8. 8
  9. 9
  10. 10
  11. 11
  12. 12
  13. 13
  14. 14
  15. 15
  16. 16
  17. 17
  18. 18
  19. 19
  20. 20
  21. 21
  22. 22
  23. 23
  24. 24
  25. 25
  26. 26
  27. 27
  28. 28
  29. 29
  30. 30
  31. 31
  32. 32
  33. 33
  34. 34
  35. 35
  36. 36
  37. 37
  38. 38
  39. 39
  40. 40
  41. 41
  42. 42
  43. 43
  44. 44
  45. 45
  46. 46
  47. 47
  48. 48

Information

Approved for 5 Category 1 SA Credits on 12/1/24 through 12/1/25
Learning Objectives: Osgood-Schlatter Disease, Osteochondritis dissecans, Pediatric Monteggia fracture, Osteochondroma, Ankle Sprain or Salter-Harris Injury, Toddler’s Fracture, Legg-Calve-Perthes Disease (LCPD), Metatarsal Stress Fractures, Pediatric Ankle Fracture, Pediatric Elbow Trauma, elbow ocd, plastic deformation, patellar dislocation, Tillaux fracture, Shin splints, Non accidental trauma, pediatric proximal humerus, Asymptomatic OCD of the Knee, Adolescent Idiopathic Scoliosis with Left Curve, Os peroneum, Os vesalianum, Osteochondroma, Patella retinacular tear, Juvenile Hallux Valgus, Predicting Progression of Adolescent Idiopathic Scoliosis, Kohler’s Disease, Tibial Spine Avulsion Fracture, Severs disease, Pediatric Triplane Fracture, Slipped Capital Femoral Epiphysis (SCFE), Pediatric Ankle Fractures, Lateral Tibial Plateau Fracture, Spondylolysis, Right Femur Fracture, Snapping Hip, Less Trochanter Avulsion Fracture, and a Supracondylar Process

You have already completed the quiz before. Hence you can not start it again.

Quiz is loading...

You must sign in or sign up to start the quiz.

You have to finish following quiz, to start this quiz:

Results

0 of 48 questions answered correctly

Your time:

Time has elapsed

You have reached 0 of 0 points, (0)

Categories

  1. Not categorized 0%
  • You are required to complete the post-exam evaluation to receive CME credits. Please complete the post-exam evaluation here. Your CME certificate will be emailed to you shortly. If you don’t receive your certificate today, and the certificate email from CME4PAC is not located in your spam folder, please email, Dagan Cloutier, at dcloutier@cme4pac.com.

  1. 1
  2. 2
  3. 3
  4. 4
  5. 5
  6. 6
  7. 7
  8. 8
  9. 9
  10. 10
  11. 11
  12. 12
  13. 13
  14. 14
  15. 15
  16. 16
  17. 17
  18. 18
  19. 19
  20. 20
  21. 21
  22. 22
  23. 23
  24. 24
  25. 25
  26. 26
  27. 27
  28. 28
  29. 29
  30. 30
  31. 31
  32. 32
  33. 33
  34. 34
  35. 35
  36. 36
  37. 37
  38. 38
  39. 39
  40. 40
  41. 41
  42. 42
  43. 43
  44. 44
  45. 45
  46. 46
  47. 47
  48. 48
  1. Answered
  2. Review
  1. Question 1 of 48
    1. Question

    A 17 year-old male presents to the office with right elbow pain for several months. He plays baseball 7 months out of the year and either pitches or plays third base. His pain is made worse when throwing and he has lost velocity on his fastball. Ice and NSAIDs provide little relief. He tried taking a few months off from pitching but the pain returned as he started throwing again. On exam you note tenderness over the radiocapitellar joint and pain during the lateral compression test. He has no medial sided laxity or pain with valgus stress. AP and lateral x-rays of the right elbow are normal. What is the most likely diagnosis?

    Correct

    The overhead throwing motion places significant stress on the elbow which can often lead to overuse injury. Determining pathology in the overhead athlete starts with the basic understanding that a valgus force is placed on the elbow during the throwing motion. This causes tensile forces to the medial elbow and compression forces to the lateral elbow. Common sources of medial elbow pain in the throwing athlete are often attributed to a stretch of the medial structures including ulnar collateral ligament tearing, ulnar neuritis, flexor pronator strain, and medial apophysitis. Repetitive compressive forces at the lateral elbow may cause a condition called osteochondritis dissecans (OCD) of the capitellum. 1
    OCD of the capitellum occurs when repetitive trauma starts to damage the articular cartilage and subchondral bone of the capitellum. The condition generally occurs between the ages of 11 and 21 and is more common in males. Pain is often described as worse with activities and better with rest, with the occasional catching or locking occurring during throwing motion. On exam tenderness along the radiocapitellar articulation and pain with the lateral compression test is often noted. Radiographs are generally negative early in the disease but may show irregularity to the articular surface as the disease advances. MRI is the most sensitive study for picking up OCD early. Early stable OCD can be treated successfully with 3-6 weeks of rest followed by a gradual return to throwing. Large or unstable OCD lesions often require operative debridement and lesion excision, debridement and bone marrow stimulation, or fragment fixation. (Note: Panner disease occurs almost exclusively in males under 10 years of age and is not associated with trauma.) 1,2
    References
    1. Patel RM, Lynch TS, Amin NH, Gryzlo S, Schickenantz M. Elbow Injuries in the Throwing Athlete. JBJS Reviews 2014; 2 (11). Pages 1-11.
    2. Ruchelsman DE, Hall MP, Youm T. Osteochondritis Dissecans of the Capitellum: Current Concepts. JAAOS 2010; 18: 557-567.

    Incorrect

    The overhead throwing motion places significant stress on the elbow which can often lead to overuse injury. Determining pathology in the overhead athlete starts with the basic understanding that a valgus force is placed on the elbow during the throwing motion. This causes tensile forces to the medial elbow and compression forces to the lateral elbow. Common sources of medial elbow pain in the throwing athlete are often attributed to a stretch of the medial structures including ulnar collateral ligament tearing, ulnar neuritis, flexor pronator strain, and medial apophysitis. Repetitive compressive forces at the lateral elbow may cause a condition called osteochondritis dissecans (OCD) of the capitellum. 1
    OCD of the capitellum occurs when repetitive trauma starts to damage the articular cartilage and subchondral bone of the capitellum. The condition generally occurs between the ages of 11 and 21 and is more common in males. Pain is often described as worse with activities and better with rest, with the occasional catching or locking occurring during throwing motion. On exam tenderness along the radiocapitellar articulation and pain with the lateral compression test is often noted. Radiographs are generally negative early in the disease but may show irregularity to the articular surface as the disease advances. MRI is the most sensitive study for picking up OCD early. Early stable OCD can be treated successfully with 3-6 weeks of rest followed by a gradual return to throwing. Large or unstable OCD lesions often require operative debridement and lesion excision, debridement and bone marrow stimulation, or fragment fixation. (Note: Panner disease occurs almost exclusively in males under 10 years of age and is not associated with trauma.) 1,2
    References
    1. Patel RM, Lynch TS, Amin NH, Gryzlo S, Schickenantz M. Elbow Injuries in the Throwing Athlete. JBJS Reviews 2014; 2 (11). Pages 1-11.
    2. Ruchelsman DE, Hall MP, Youm T. Osteochondritis Dissecans of the Capitellum: Current Concepts. JAAOS 2010; 18: 557-567.

  2. Question 2 of 48
    2. Question

    A 7 year-old male presents to emergency department with a left arm injury that occurred a few hours earlier. He was riding on a scooter and fell forward landing on his outstretched hand. On exam you note a slight angular deformity to the left forearm. AP and lateral x-rays show a distal 1/3 radius fracture with plastic deformation (figures 1 and 2). Which choice below is true regarding pediatric bone and plastic deformation?

    Correct

    Plastic deformation, or traumatic bowing, is an irreversible change in the shape of bone after an injury which occurs almost exclusively in pediatric forearm fractures. Plastic deformation occurs when a large amount of force is placed on bone and the bone stays in the deformed position (plasticity) even after the force is removed. The modulus of elasticity, or the measure of stiffness of a solid material, is higher in adult bone than pediatric bone. (Another example of modulus of elasticity: metal implants used in orthopedic surgery have a higher modulus of elasticity than bone). Mineral content of bone plays a large role in its elastic properties. Pediatric corticol bone has lower mineral content than adult bone and therefore has weaker bending strength. With aging, the increased mineralization stiffens the collagen and hydroxyapatite in bone, which decreases the amount of deformity the bone will tolerate before fracture. Unlike adult bone, pediatric bone is able to absorb significant bending forces due to plastic deformation properties prior to fracturing. 1,2
    A plastic deformity of the forearm can limit forearm supination and pronation as the deformed radius and ulna encroach upon the interosseous space. In general, plastic deformities are thought to have less remodeling potential and can be more difficult to reduce than other angulated pediatric forearm fractures. Therefore the treatment approach is unique as the threshold for closed reduction under anesthesia is less. One recommendation includes close reducing plastic deformities under general anesthesia with greater than 10 degrees of angulation in all patients over 6 years of age. In general, displaced forearm fractures (non-plastic deformity) generally tolerate 15-20 degrees of angulation before closed reduction is attempted in patients less than 10 years of age. 1,2

    References
    1. Mabrey JD, Fitch RD. Deformation in pediatric fractures: mechanism and treatment. Pediatr Orthop. 1989 May-Jun;9(3):310-4.
    2. Wall EJ, Mehlman CT. Injuries to the Shafts of the Radius and Ulna. Rockwood and Wilkins’ Fractures in Children 6th Edition. 2001 Philadelpia, PA. Pg 417-418

    Incorrect

    Plastic deformation, or traumatic bowing, is an irreversible change in the shape of bone after an injury which occurs almost exclusively in pediatric forearm fractures. Plastic deformation occurs when a large amount of force is placed on bone and the bone stays in the deformed position (plasticity) even after the force is removed. The modulus of elasticity, or the measure of stiffness of a solid material, is higher in adult bone than pediatric bone. (Another example of modulus of elasticity: metal implants used in orthopedic surgery have a higher modulus of elasticity than bone). Mineral content of bone plays a large role in its elastic properties. Pediatric corticol bone has lower mineral content than adult bone and therefore has weaker bending strength. With aging, the increased mineralization stiffens the collagen and hydroxyapatite in bone, which decreases the amount of deformity the bone will tolerate before fracture. Unlike adult bone, pediatric bone is able to absorb significant bending forces due to plastic deformation properties prior to fracturing. 1,2
    A plastic deformity of the forearm can limit forearm supination and pronation as the deformed radius and ulna encroach upon the interosseous space. In general, plastic deformities are thought to have less remodeling potential and can be more difficult to reduce than other angulated pediatric forearm fractures. Therefore the treatment approach is unique as the threshold for closed reduction under anesthesia is less. One recommendation includes close reducing plastic deformities under general anesthesia with greater than 10 degrees of angulation in all patients over 6 years of age. In general, displaced forearm fractures (non-plastic deformity) generally tolerate 15-20 degrees of angulation before closed reduction is attempted in patients less than 10 years of age. 1,2

    References
    1. Mabrey JD, Fitch RD. Deformation in pediatric fractures: mechanism and treatment. Pediatr Orthop. 1989 May-Jun;9(3):310-4.
    2. Wall EJ, Mehlman CT. Injuries to the Shafts of the Radius and Ulna. Rockwood and Wilkins’ Fractures in Children 6th Edition. 2001 Philadelpia, PA. Pg 417-418

  3. Question 3 of 48
    3. Question


    An 18 year-old male presents to the office one week after sustaining a patella dislocation while playing football. He admits to falling awkwardly and noticing a deformity to his knee cap. He was able to reduce the patella with his hand and was able to walk after the reduction. He now presents with swelling and significant pain when flexing his right knee. Sunrise view x-ray (figure 1) shows a reduced patella located centrally within the trochlea groove and a small avulsion type fracture off the medial patella. What is the next best step in this patient’s treatment?

    Correct

    The patient presents with swelling and pain after a patella dislocation. Given his swelling and x-ray findings, an MRI should be ordered to rule out an osteochondral defect and injury to the medial patellofemoral ligament (MPFL). A traumatic patella dislocation can break off an osteochondral fragment, generally off the lateral femoral condyle as the patella dislocates laterally. An osteochondral fragment can break off and float around the knee causing symptoms of a loose body. Symptoms include recurrent effusions and mechanical symptoms of catching or locking of the knee. The incidence of loose body formation after patella dislocation was found to be 22% in one study. 1
    MPFL disruption may occur in up to 96% of patients that sustain a traumatic patella dislocation. The MPFL is the primary soft tissue stabilizer to lateral dislocation of the patella. The MPFL also helps the patella stay located centrally within the trochlea during knee flexion. MRI is the study of choice to determine if the MPFL is intact after a traumatic patella dislocation with a sensitivity of 85% and an accuracy of 80%. The standard of care for patella dislocations with MPFL disruption and without an OCD lesion is non-operative treatment. Conservative treatment is usually successfully with redislocation rates ranging from 15% to 44%. 2

    References
    1. Farr J, Covell DJ, Lattermann C. Cartilage lesions in patellofemoral dislocations: Incidents/locations/when to treat. Sports Med Arthrosc. 2012 Sep; 20(3): 181–186.
    2. Amin NH, Lynch TS, Patel RM, Patel N, Saluan P. Medial Patellafemoral ligament reconstruction. JBJS Reviews 2015; 3 (7): e3

    Incorrect

    The patient presents with swelling and pain after a patella dislocation. Given his swelling and x-ray findings, an MRI should be ordered to rule out an osteochondral defect and injury to the medial patellofemoral ligament (MPFL). A traumatic patella dislocation can break off an osteochondral fragment, generally off the lateral femoral condyle as the patella dislocates laterally. An osteochondral fragment can break off and float around the knee causing symptoms of a loose body. Symptoms include recurrent effusions and mechanical symptoms of catching or locking of the knee. The incidence of loose body formation after patella dislocation was found to be 22% in one study. 1
    MPFL disruption may occur in up to 96% of patients that sustain a traumatic patella dislocation. The MPFL is the primary soft tissue stabilizer to lateral dislocation of the patella. The MPFL also helps the patella stay located centrally within the trochlea during knee flexion. MRI is the study of choice to determine if the MPFL is intact after a traumatic patella dislocation with a sensitivity of 85% and an accuracy of 80%. The standard of care for patella dislocations with MPFL disruption and without an OCD lesion is non-operative treatment. Conservative treatment is usually successfully with redislocation rates ranging from 15% to 44%. 2

    References
    1. Farr J, Covell DJ, Lattermann C. Cartilage lesions in patellofemoral dislocations: Incidents/locations/when to treat. Sports Med Arthrosc. 2012 Sep; 20(3): 181–186.
    2. Amin NH, Lynch TS, Patel RM, Patel N, Saluan P. Medial Patellafemoral ligament reconstruction. JBJS Reviews 2015; 3 (7): e3

  4. Question 4 of 48
    4. Question


    A 17 year-old male presents to the office with severe right ankle pain after a twisting injury at a trampoline park. AP, mortise, and lateral x-rays of the right ankle are shown in figures 1, 2, and 3 respectively. What is the name of this fracture?

    Correct

    This patient has a Tillaux fracture. A Tillaux fracture occurs with an abduction-external rotation mechanism which causes the anterior tibiofibular ligament to avulse the anterolateral corner of the distal tibial epiphysis. The fracture is a Salter-Harris III fracture of the anterolateral distal tibia epiphysis. The fracture occurs in patients nearing skeletal maturity and is more common in girls. Standard AP, lateral, and mortise view x-rays should be obtained in all pediatric patients with ankle injuries. A Tillaux fracture appears as a vertical line through the epiphysis which is best seen on mortise view. CT is often ordered to help determine the degree of fracture displacement. Fractures with <2 mm of displacement can be treated in a non-weight bearing cast. Closed reduction is necessary in fractures displaced > 2mm to restore the joint surface and prevent future degenerative joint disease. Displaced fractures can be reduced with an internal rotation of the foot maneuver. Surgical fixation includes the placement of one or two screws across the epiyphysis to hold the reduction in place. A triplane fracture is a multi-plane Salter-Harris IV fracture. A triplance fracture has a coronal plane fracture in the posterior distal tibial metaphysis which distinguishes it from a Tillaux fracture. 1,2
    Answer C.
    References
    1. Shital P, Wells L, Mehlman C, Scherl S. Management Of Fractures In Adolescents. The Journal Of Bone & Joint Surgery – Selected Instructional Course Lecture: 15 December 2010 – Volume 92 – Issue 18 – p. 2947-2958.
    2. Tillaux fracture. http://www.orthobullets.com. Accessed on 12/2/2018.

    Incorrect

    This patient has a Tillaux fracture. A Tillaux fracture occurs with an abduction-external rotation mechanism which causes the anterior tibiofibular ligament to avulse the anterolateral corner of the distal tibial epiphysis. The fracture is a Salter-Harris III fracture of the anterolateral distal tibia epiphysis. The fracture occurs in patients nearing skeletal maturity and is more common in girls. Standard AP, lateral, and mortise view x-rays should be obtained in all pediatric patients with ankle injuries. A Tillaux fracture appears as a vertical line through the epiphysis which is best seen on mortise view. CT is often ordered to help determine the degree of fracture displacement. Fractures with <2 mm of displacement can be treated in a non-weight bearing cast. Closed reduction is necessary in fractures displaced > 2mm to restore the joint surface and prevent future degenerative joint disease. Displaced fractures can be reduced with an internal rotation of the foot maneuver. Surgical fixation includes the placement of one or two screws across the epiyphysis to hold the reduction in place. A triplane fracture is a multi-plane Salter-Harris IV fracture. A triplance fracture has a coronal plane fracture in the posterior distal tibial metaphysis which distinguishes it from a Tillaux fracture. 1,2
    Answer C.
    References
    1. Shital P, Wells L, Mehlman C, Scherl S. Management Of Fractures In Adolescents. The Journal Of Bone & Joint Surgery – Selected Instructional Course Lecture: 15 December 2010 – Volume 92 – Issue 18 – p. 2947-2958.
    2. Tillaux fracture. http://www.orthobullets.com. Accessed on 12/2/2018.

  5. Question 5 of 48
    5. Question


    A 16 year-old girl presents to the office with 2 months of lower leg pain. The pain seems to be located over the distal medial tibia on both legs. She dances up to 3 hours a day which aggravates her pain. Figures 1 and 2 show the alignment of her lower extremities and flat foot deformities bilaterally. Which skeletal malalignment is most likely contributing to this patient’s pain?

    Correct

    Overuse injuries are increasingly common in today’s society as kids practice for hours each day in the same sport throughout the year. Exercise induced pain in the lower extremities is often seen in teenage athletes. Lower extremity pain most commonly occurs from either a tibial stress fracture, medial tibial stress syndrome, or chronic compartment syndrome. Medial tibial stress syndrome, or shin splints, is the most common of the three. Pain from medial tibial stress syndrome is located at the mid to distal medial tibia. The pain is thought to arise from traction periostitis of the flexor digitorum longus and soleus muscles as they attach on the medial tibia. Medial tibial stress syndrome is often called soleus syndrome for this reason. The most common skeletal malalignment thought to contribute to medial tibial stress syndrome is excessive pronation of the foot. Excessive pronation of the foot strains the soleus muscle which acts to resist pronation. Conservative treatment with a period of rest and/or a decrease in training intensity is the first line treatment. Orthotics can help support a flat foot deformity. 1,2
    Answer A.
    References
    1. Yates Ben, FCPod(S); Allen Mike J., FRCS; Barnes Mike R., BSc;Outcome Of Surgical Treatment Of Medial Tibial Stress Syndrome. The Journal Of Bone & Joint Surgery – Scientific Article: 00 October 2003 – Volume 85 – Issue 10 – p. 1974-1980
    2. Tibial Stress Syndrome. http://www.orthobullets.com. Accessed on 9/21/18.

    Incorrect

    Overuse injuries are increasingly common in today’s society as kids practice for hours each day in the same sport throughout the year. Exercise induced pain in the lower extremities is often seen in teenage athletes. Lower extremity pain most commonly occurs from either a tibial stress fracture, medial tibial stress syndrome, or chronic compartment syndrome. Medial tibial stress syndrome, or shin splints, is the most common of the three. Pain from medial tibial stress syndrome is located at the mid to distal medial tibia. The pain is thought to arise from traction periostitis of the flexor digitorum longus and soleus muscles as they attach on the medial tibia. Medial tibial stress syndrome is often called soleus syndrome for this reason. The most common skeletal malalignment thought to contribute to medial tibial stress syndrome is excessive pronation of the foot. Excessive pronation of the foot strains the soleus muscle which acts to resist pronation. Conservative treatment with a period of rest and/or a decrease in training intensity is the first line treatment. Orthotics can help support a flat foot deformity. 1,2
    Answer A.
    References
    1. Yates Ben, FCPod(S); Allen Mike J., FRCS; Barnes Mike R., BSc;Outcome Of Surgical Treatment Of Medial Tibial Stress Syndrome. The Journal Of Bone & Joint Surgery – Scientific Article: 00 October 2003 – Volume 85 – Issue 10 – p. 1974-1980
    2. Tibial Stress Syndrome. http://www.orthobullets.com. Accessed on 9/21/18.

  6. Question 6 of 48
    6. Question


    A 6 month-old presents to the emergency department with left leg pain and contusions about the head and torso. AP and lateral x-rays of the left tibia show a displaced left tibial shaft fracture. Child abuse is suspected with the patient’s injury pattern. Which clinical finding below is NOT a sign of suspected child abuse?

    Correct

    Child abuse is the 4th leading cause of childhood death in the United States and an unfortunate reality in the emergency room setting. Detecting child abuse is critical to ensure the child is removed from an abusive setting and transitioned to a safe environment. In some circumstances, orthopedic providers may be the first to recognize suspected abuse based on patient history and injury pattern. Historical findings that may indicate child abuse include delay in presenting, a vague history of injury, history of fall, and insufficient mechanism of injury explained to support injury. Characteristic of a child at higher risk of abuse include less than 3 years old, behavioral problems, stepchild, and low socioeconomic status. Fracture patterns suspected of child abuse include rip fractures, multiple fractures, fractures in various stages of healing, a long bone fracture in a child <2 years old, vertebral compression fractures, and metaphyseal corner fractures. A metaphyseal corner fracture is characterized by a small piece of bone avulsed off the metaphysis of long bones. The injury is caused by a child under 2 years old be shaken by the torso which causes whiplash or shear forces to long bones of the extremities.  1,2

    Answer A.

    References

    1. Kocker MS, Kasser JR. Orthopedic Aspects of Child Abuse. J Am Acad Orthop Surg 2000;8:10-20
    2. Guenther E, Knight S, Olson LM, Dean M, Keenan HT. Prediction of Child Abuse Risk from Emergency Department Use. J Pediatr. 2009 Feb; 154(2): 272–277.
    Incorrect

    Child abuse is the 4th leading cause of childhood death in the United States and an unfortunate reality in the emergency room setting. Detecting child abuse is critical to ensure the child is removed from an abusive setting and transitioned to a safe environment. In some circumstances, orthopedic providers may be the first to recognize suspected abuse based on patient history and injury pattern. Historical findings that may indicate child abuse include delay in presenting, a vague history of injury, history of fall, and insufficient mechanism of injury explained to support injury. Characteristic of a child at higher risk of abuse include less than 3 years old, behavioral problems, stepchild, and low socioeconomic status. Fracture patterns suspected of child abuse include rip fractures, multiple fractures, fractures in various stages of healing, a long bone fracture in a child <2 years old, vertebral compression fractures, and metaphyseal corner fractures. A metaphyseal corner fracture is characterized by a small piece of bone avulsed off the metaphysis of long bones. The injury is caused by a child under 2 years old be shaken by the torso which causes whiplash or shear forces to long bones of the extremities.  1,2

    Answer A.

    References

    1. Kocker MS, Kasser JR. Orthopedic Aspects of Child Abuse. J Am Acad Orthop Surg 2000;8:10-20
    2. Guenther E, Knight S, Olson LM, Dean M, Keenan HT. Prediction of Child Abuse Risk from Emergency Department Use. J Pediatr. 2009 Feb; 154(2): 272–277.
  7. Question 7 of 48
    7. Question

    A 14 year-old male presents to your office with left shoulder pain after a fall two days prior. He is unable to lift the arm to the side or forward. AP and lateral x-rays of the left shoulder are shown in figures 1 and 2. What is the best treatment option?

     

    Correct

    Proximal humerus fractures account for <5% of fractures in children and the peak age of occurrence is 15 years old. Nearly all proximal humerus and midshaft humerus fractures in the pediatric population can be treated nonoperatively with good results. The younger the patient the greater potential for bone remodeling and greater degree of fracture displacement that the body will correct over time. Physeal closure between the humeral head and the shaft occurs at ages 16 to 19 years of age. Once the physis closes remodeling is not likely to occur. There is no widely accepted criteria to what amount of fracture displacement is acceptable for nonoperative treatment. One age dependent guideline allowed for up to 75 degrees of angulation for patients under 7 years old, up to 60 degrees from 8-11 years old, and up to 45 degrees in children over 12 years old. A hanging arm cast is a common treatment modality that allows for passive fracture reduction as gravity provides traction on the humerus. The long arm hanging cast shown below is generally continued for 3-4 weeks or until early healing can be seen on x-ray. In older pediatric patients with significant fracture displacement, closed reduction and percutaneous pinning may be considered as minimally invasive option. 1, 2

    References
    1.Bahrs C, Zipplies S, Ochs, BG, Rether J, Oehms J, Eingarttner C, Rolauffs B, Weise K. Proximal Humeral Fractures in Children and Adolescents. Journal of Pediatric Orthopaedics. 2009; 29(3):238-242.
    2.Baxter MP, Wiley JJ. Fractures of the Proximal Humeral Epiphysis. Journal of Bone and Joint Surgery. 1986; Aug 4:570-573.

    Incorrect

    Proximal humerus fractures account for <5% of fractures in children and the peak age of occurrence is 15 years old. Nearly all proximal humerus and midshaft humerus fractures in the pediatric population can be treated nonoperatively with good results. The younger the patient the greater potential for bone remodeling and greater degree of fracture displacement that the body will correct over time. Physeal closure between the humeral head and the shaft occurs at ages 16 to 19 years of age. Once the physis closes remodeling is not likely to occur. There is no widely accepted criteria to what amount of fracture displacement is acceptable for nonoperative treatment. One age dependent guideline allowed for up to 75 degrees of angulation for patients under 7 years old, up to 60 degrees from 8-11 years old, and up to 45 degrees in children over 12 years old. A hanging arm cast is a common treatment modality that allows for passive fracture reduction as gravity provides traction on the humerus. The long arm hanging cast shown below is generally continued for 3-4 weeks or until early healing can be seen on x-ray. In older pediatric patients with significant fracture displacement, closed reduction and percutaneous pinning may be considered as minimally invasive option. 1, 2

    References
    1.Bahrs C, Zipplies S, Ochs, BG, Rether J, Oehms J, Eingarttner C, Rolauffs B, Weise K. Proximal Humeral Fractures in Children and Adolescents. Journal of Pediatric Orthopaedics. 2009; 29(3):238-242.
    2.Baxter MP, Wiley JJ. Fractures of the Proximal Humeral Epiphysis. Journal of Bone and Joint Surgery. 1986; Aug 4:570-573.

  8. Question 8 of 48
    8. Question

    A 14 year old male presents to the office with left knee pain after playing basketball 2 days ago. He is diagnosed with a grade I medial collateral ligament (MCL) sprain but a large 1.6 cm osteochondral (OCD) lesion on the medial femoral condyle is found incidentally on an AP x-ray (figure 1). Coronal MRI of the knee shows intact articular cartilage with no reactive edema around the OCD lesion. What is the best treatment option?

    Correct

    The patient has a large asymptomatic osteochondral lesion of the medial femoral condyle. MRI shows intact articular cartilage over the OCD lesion and no increase signal intensity in the surrounding bone which represents a stable lesion. This lesion was found incidentally on x-ray and is not causing the patients symptoms. Skeletally immature (juvenile) patients with an open physis and a stable lesion usually heal with non-operative treatment. Stable OCD lesions often become asymptomatic long before bone marrow edema subsides on MRI and x-rays shows signs of healing. For that reason, asymptomatic patients with stable lesions are usually not followed with serial x-rays or additional MRI. Skeletally immature patients that are symptomatic and have unstable lesions are unlikely to heal with conservative treatment and usually require surgery to prevent early onset arthritis. Osteochondral lesions in adults usually require operative treatment to prevent detachment of the OCD lesion and resulting early onset arthritis. Restricting running and jumping activities in juvenile patients with asymptomatic lesions has not been shown to improve outcomes. 1,2
    Answer A
    References
    1. Eric J. Wall, MD,1 Jason Vourazeris, BS,1 Gregory D. Myer, MS, CSCS,1 Kathleen H. Emery, MD,1 Jon G. Divine, MD,1 Todd G. Nick, PhD,1 and Timothy E. Hewett, PhD1 The Healing Potential of Stable Juvenile Osteochondritis Dissecans Knee Lesions. J Bone Joint Surg Am. 2008 Dec 1; 90(12): 2655–2664
    2. Osteochondral lesions. http://www.orthobullets.com. Accessed on 12/22/2019.

    Incorrect

    The patient has a large asymptomatic osteochondral lesion of the medial femoral condyle. MRI shows intact articular cartilage over the OCD lesion and no increase signal intensity in the surrounding bone which represents a stable lesion. This lesion was found incidentally on x-ray and is not causing the patients symptoms. Skeletally immature (juvenile) patients with an open physis and a stable lesion usually heal with non-operative treatment. Stable OCD lesions often become asymptomatic long before bone marrow edema subsides on MRI and x-rays shows signs of healing. For that reason, asymptomatic patients with stable lesions are usually not followed with serial x-rays or additional MRI. Skeletally immature patients that are symptomatic and have unstable lesions are unlikely to heal with conservative treatment and usually require surgery to prevent early onset arthritis. Osteochondral lesions in adults usually require operative treatment to prevent detachment of the OCD lesion and resulting early onset arthritis. Restricting running and jumping activities in juvenile patients with asymptomatic lesions has not been shown to improve outcomes. 1,2
    Answer A
    References
    1. Eric J. Wall, MD,1 Jason Vourazeris, BS,1 Gregory D. Myer, MS, CSCS,1 Kathleen H. Emery, MD,1 Jon G. Divine, MD,1 Todd G. Nick, PhD,1 and Timothy E. Hewett, PhD1 The Healing Potential of Stable Juvenile Osteochondritis Dissecans Knee Lesions. J Bone Joint Surg Am. 2008 Dec 1; 90(12): 2655–2664
    2. Osteochondral lesions. http://www.orthobullets.com. Accessed on 12/22/2019.

  9. Question 9 of 48
    9. Question

    A 10 year old female presents to your office accompanied by her parents with complaints of upper back asymmetry. She denies any pain or any noticeable functional limitations. Her father admits to a history of scoliosis. On a forward bend test you notice left flank and right rib prominence. She walks with a normal gait and has no motor weakness in the lower extremities. Figure 1 shows a scoliosis view x-ray with a left coronal Cobb angle of 27 degrees (T9-L3). What is the next best step in treatment?

    Correct

    Adolescent idiopathic scoliosis is a lateral bend to the spine without an identifiable cause and usually occurs between the ages of 10 and 18 years old. The lateral bend is measured as a Cobb angle which should be >10 degrees to diagnose scoliosis. Scoliosis is further defined by right of left based on the direction of the apical convexity. In adolescents, right thoracic curves are the most common and left thoracic curves are rare. When a left thoracic scoliosis is present MRI should be obtained to rule out spinal cord abnormalities as a possible cause. MRI should show the entire spinal cord from the posterior fossa of the skull to the conus medullaris. Such abnormalities may include a Chiari malformation, syringomyelia, and/or a tethered cord. Another predictor of spinal cord abnormalities causing scoliosis is curve severity with curves greater than 40 degrees more likely to have an abnormality. Spinal cord abnormalities causing left sided scoliosis are more common in males where right sided “idiopathic” scoliosis is more common in females. Genetics seems to play the largest role in the development of scoliosis. If both parents had idiopathic adolescent scoliosis their children are 50 times more likely to get it than the general population. 1,2
    Answer B.
    References
    1. Wu L1, Qiu Y, Wang B, Zhu ZZ, Ma WW. The left thoracic curve pattern: a strong predictor for neural axis abnormalities in patients with “idiopathic” scoliosis. Spine (Phila Pa 1976). 2010 Jan 15;35(2):182-5.
    2. Reamy, Brian V., and Joseph Slakey. “Adolescent idiopathic scoliosis: review and current concepts.” American family physician 64.1 (2001): 111.

    Incorrect

    Adolescent idiopathic scoliosis is a lateral bend to the spine without an identifiable cause and usually occurs between the ages of 10 and 18 years old. The lateral bend is measured as a Cobb angle which should be >10 degrees to diagnose scoliosis. Scoliosis is further defined by right of left based on the direction of the apical convexity. In adolescents, right thoracic curves are the most common and left thoracic curves are rare. When a left thoracic scoliosis is present MRI should be obtained to rule out spinal cord abnormalities as a possible cause. MRI should show the entire spinal cord from the posterior fossa of the skull to the conus medullaris. Such abnormalities may include a Chiari malformation, syringomyelia, and/or a tethered cord. Another predictor of spinal cord abnormalities causing scoliosis is curve severity with curves greater than 40 degrees more likely to have an abnormality. Spinal cord abnormalities causing left sided scoliosis are more common in males where right sided “idiopathic” scoliosis is more common in females. Genetics seems to play the largest role in the development of scoliosis. If both parents had idiopathic adolescent scoliosis their children are 50 times more likely to get it than the general population. 1,2
    Answer B.
    References
    1. Wu L1, Qiu Y, Wang B, Zhu ZZ, Ma WW. The left thoracic curve pattern: a strong predictor for neural axis abnormalities in patients with “idiopathic” scoliosis. Spine (Phila Pa 1976). 2010 Jan 15;35(2):182-5.
    2. Reamy, Brian V., and Joseph Slakey. “Adolescent idiopathic scoliosis: review and current concepts.” American family physician 64.1 (2001): 111.

  10. Question 10 of 48
    10. Question

    A 53 year-old male presents to the office with a 5 month history of right ankle pain. He believes the pain started after twisting his ankle while dancing at a wedding. The pain was sharp in nature but he was able to continue dancing that night. The pain improved slowly but he continues to complain of occasional sharp pain in the lateral foot with physical activities. An oblique view x-ray of the foot (figure 1) taken at the emergency department is read as a likely avulsion fracture of the lateral cuboid. What is the best treatment option for this patient?

    Correct

    The oblique view x-ray of the right foot shows a well corticated oval shaped bone lateral to the calcaneocuboid joint which is consistent with an os peroneum. Accessory ossicles like an os peroneum are often confused with avulsion fracture. An os peroneum is an accessory ossicle imbedded within the peroneus longus tendon. The peroneus longus tendon courses from the posterior fibula to the lateral calcaneus and cuboid before inserting into the first metatarsal and medial cuneiform. An os peroneum is found in 20% of the population and occurs bilaterally in 60% of cases. The ossicle is usually asymptomatic but can become painful with an injury. A painful os peroneum, often called os peroneum syndrome, can be due to a fractured ossicle and tenosynovitis or tearing of the peroneus longus tendon. The most common physical exam finding is pain to palpation over the ossicle. There are many potential causes of lateral foot pain so an MRI is the best study to evaluate for the broad differential diagnosis. Treatment of os peroneum syndrome includes rest, ice, oral NSAIDS and immobilization with a walking boot or cast. Operative treatment of a painful os peroneum includes excision of the ossicle with primary repair of the peroneus longus tendon. 1,2
    Answer B
    References
    1. Heckman DS, Reddy S, Pedowitz D, Wapner KL, Parekh SG. Operative Treatment For Peroneal Tendon Disorders. The Journal Of Bone & Joint Surgery – Current Concepts Review: 01 February 2008 – Volume 90 – Issue 2 – p. 404-418
    2. Vora B, Wong B. Common accessory ossicles of the foot: imaging features, pitfalls and associated pathology. Singapore Med J. 2018 Apr; 59(4): 183–189.

    Incorrect

    The oblique view x-ray of the right foot shows a well corticated oval shaped bone lateral to the calcaneocuboid joint which is consistent with an os peroneum. Accessory ossicles like an os peroneum are often confused with avulsion fracture. An os peroneum is an accessory ossicle imbedded within the peroneus longus tendon. The peroneus longus tendon courses from the posterior fibula to the lateral calcaneus and cuboid before inserting into the first metatarsal and medial cuneiform. An os peroneum is found in 20% of the population and occurs bilaterally in 60% of cases. The ossicle is usually asymptomatic but can become painful with an injury. A painful os peroneum, often called os peroneum syndrome, can be due to a fractured ossicle and tenosynovitis or tearing of the peroneus longus tendon. The most common physical exam finding is pain to palpation over the ossicle. There are many potential causes of lateral foot pain so an MRI is the best study to evaluate for the broad differential diagnosis. Treatment of os peroneum syndrome includes rest, ice, oral NSAIDS and immobilization with a walking boot or cast. Operative treatment of a painful os peroneum includes excision of the ossicle with primary repair of the peroneus longus tendon. 1,2
    Answer B
    References
    1. Heckman DS, Reddy S, Pedowitz D, Wapner KL, Parekh SG. Operative Treatment For Peroneal Tendon Disorders. The Journal Of Bone & Joint Surgery – Current Concepts Review: 01 February 2008 – Volume 90 – Issue 2 – p. 404-418
    2. Vora B, Wong B. Common accessory ossicles of the foot: imaging features, pitfalls and associated pathology. Singapore Med J. 2018 Apr; 59(4): 183–189.

  11. Question 11 of 48
    11. Question

    A 21 year-old male presents to the office with left foot pain after twisting his ankle 2 days prior. He was walking through the woods and stepped awkwardly on a rock which caused the foot to buckle. He has had difficulty walking on the foot since. On physical exam of the left foot he has tenderness to palpation over the proximal 5th metatarsal. He has no edema or ecchymosis noted over the lateral foot. Oblique radiograph is shown above. Which abnormality can be seen on this patient’s radiographs?

    Correct

    The proximal 5th metatarsal is divided into 3 main sections: the proximal tubercle, the metaphyseal-diaphyseal junction, and the diaphysis. Fractures at the metaphyseal-diaphyseal junction are referred to as Jones fractures. A pseudo Jones fracture is used to describe a proximal tubercle fracture. The lateral band of the plantar fascia and the peroneal brevis tendon attach to the proximal tubercle and can cause avulsion fractures with an inversion ankle injury. A fifth metatarsal apophysis can also be seen as a small sliver of bone on the lateral cortex of the 5th metatarsal, generally occurring in girls between the ages of 9 to 11 and boys 11 to 14 years of age. An proximal 5th metatarsal apophysis and accessory ossicles are often confused as small avulsion fractures. An os vesalianum is a rare accessory ossicle at the proximal 5th metatarsal and attachment site of the peroneal brevis tendon. Os peroneum is more common and occurs at the lateral boarder of the cuboid and within the peroneal brevis tendon. Accessory ossicles have rounded smooth edges where avulsion fractures have irregular scalloped edges. An os vesalianum is separated from the metatarsal by a radiolucent line of constant width and with smooth well-corticated edges, which helps distinguish the ossicle from an avulsion fracture. Base of the 5th metatarsal fractures are generally associated with edema and ecchymosis around the lateral foot.
    Answer B.
    References
    1. Dameron TB Jr. Fractures and anatomical variations of the proximal portion of the fifth metatarsal. J Bone Joint Surg [Am]. 1975;57:788–92.
    2. Kose, O. Os vesalianum pedis misdiagnosed as fifth metatarsal avulsion fracture. Emergency medicine Australasia: EMA 21(5):426 · October 2009.

    Incorrect

    The proximal 5th metatarsal is divided into 3 main sections: the proximal tubercle, the metaphyseal-diaphyseal junction, and the diaphysis. Fractures at the metaphyseal-diaphyseal junction are referred to as Jones fractures. A pseudo Jones fracture is used to describe a proximal tubercle fracture. The lateral band of the plantar fascia and the peroneal brevis tendon attach to the proximal tubercle and can cause avulsion fractures with an inversion ankle injury. A fifth metatarsal apophysis can also be seen as a small sliver of bone on the lateral cortex of the 5th metatarsal, generally occurring in girls between the ages of 9 to 11 and boys 11 to 14 years of age. An proximal 5th metatarsal apophysis and accessory ossicles are often confused as small avulsion fractures. An os vesalianum is a rare accessory ossicle at the proximal 5th metatarsal and attachment site of the peroneal brevis tendon. Os peroneum is more common and occurs at the lateral boarder of the cuboid and within the peroneal brevis tendon. Accessory ossicles have rounded smooth edges where avulsion fractures have irregular scalloped edges. An os vesalianum is separated from the metatarsal by a radiolucent line of constant width and with smooth well-corticated edges, which helps distinguish the ossicle from an avulsion fracture. Base of the 5th metatarsal fractures are generally associated with edema and ecchymosis around the lateral foot.
    Answer B.
    References
    1. Dameron TB Jr. Fractures and anatomical variations of the proximal portion of the fifth metatarsal. J Bone Joint Surg [Am]. 1975;57:788–92.
    2. Kose, O. Os vesalianum pedis misdiagnosed as fifth metatarsal avulsion fracture. Emergency medicine Australasia: EMA 21(5):426 · October 2009.

  12. Question 12 of 48
    12. Question

    A 12 year old boy presents to the office with right knee pain for the past 4 months. He describes the pain as throbbing at medial knee and worse with running. Bending the knee is particularly painful. AP x-ray of the right femur demonstrates a bony outgrowth on the medial side of the distal femur (figure 1). What is the next best step in treatment?

    Correct

    The patient’s x-ray findings are consistent with an osteochondroma, a common benign bone tumor that develops during adolescence near the surface of a growth plate. Most osteochondromas are asymptomatic and found incidentally on x-ray. Osteochondromas are made of bone and cartilage and can continue to grow up to skeletal maturity. The patient’s large osteochondroma is likely causing a mass effect on the medial soft tissues which can cause knee pain and dysfunction. This tumor is a pedunculated osteochondroma as it forms from a stalk that sticks out from normal bone. The diagnosis of an osteochondroma is often made on x-ray alone. MRI can help measure the cartilaginous cap on osteochondromas and help rule out malignant transformation, although rare. For a solitary osteochondroma, there is a 1% chance of the benign tumor transforming to a malignancy. Indications for surgical excision include pain and mechanical impingement of the tumor on soft tissues. Surgical treatment involves marginal excision of the tumor from the base of the stalk to the cartilaginous cap. The reoccurrence rate for surgical excision is less than 5%. 1,2
    Answer D.
    References
    1. August Image Quiz: Osteochondroma. Journal Of Orthopaedics For Physician Assistants – Image Quiz: 01 August 2015 – Volume 3 – Issue 3 – p. 16-17
    2. Osteochondroma. http://www.orthobullets.com. Accessed on 11/28/19.

    Incorrect

    The patient’s x-ray findings are consistent with an osteochondroma, a common benign bone tumor that develops during adolescence near the surface of a growth plate. Most osteochondromas are asymptomatic and found incidentally on x-ray. Osteochondromas are made of bone and cartilage and can continue to grow up to skeletal maturity. The patient’s large osteochondroma is likely causing a mass effect on the medial soft tissues which can cause knee pain and dysfunction. This tumor is a pedunculated osteochondroma as it forms from a stalk that sticks out from normal bone. The diagnosis of an osteochondroma is often made on x-ray alone. MRI can help measure the cartilaginous cap on osteochondromas and help rule out malignant transformation, although rare. For a solitary osteochondroma, there is a 1% chance of the benign tumor transforming to a malignancy. Indications for surgical excision include pain and mechanical impingement of the tumor on soft tissues. Surgical treatment involves marginal excision of the tumor from the base of the stalk to the cartilaginous cap. The reoccurrence rate for surgical excision is less than 5%. 1,2
    Answer D.
    References
    1. August Image Quiz: Osteochondroma. Journal Of Orthopaedics For Physician Assistants – Image Quiz: 01 August 2015 – Volume 3 – Issue 3 – p. 16-17
    2. Osteochondroma. http://www.orthobullets.com. Accessed on 11/28/19.

  13. Question 13 of 48
    13. Question

    A 10 year-old girl presents to your clinic with a history of a left foot deformity. The deformity has become noticeable over the last few years. She notices some discomfort in the foot when wearing soccer cleats. When looking at the patient’s feet from above she has a small bunion deformity bilaterally. Which statement is true regarding juvenile hallux valgus?

    Correct

    A bunion deformity, or hallux valgus, is a deformity of the first metatarsophalangeal joint that cause a prominence of the first metatarsal head medially. The deformity starts when the first phalanx deviates in a valgus position causing a varus deviation of the first metatarsal. The deformity is usually progressive and the big toe can ultimately start to angle toward, and cross over, the second toe. 1,2
    A bunion deformity is commonly seen in adults but can also occur in adolescents. The deformity in adults is usually caused by tight fitting shoes where the deformity in adolescents is usually caused by other factors. Pes planus, a long first metatarsal, metatarsus adductus, and positive family history are associated with developing a bunion in adolescence. Adolescent bunions generally occur in girls (88%) between the ages of 10 and 15 years old. MTP joint pain and stiffness is uncommon with adolescent bunions, unlike the adult form. Treatment of adolescent bunions includes night time bracing, shoe modification, and activity modification. A combination anatomical variants and open growth plates in adolescents contributes to a high rate of recurrence after surgical treatment, which is unlike the adult form. Surgical treatment is generally reserved for patients after skeletal maturity due to the high rate of postoperative complications in children. 1,2
    Answer C
    References
    1. Chell J, Dhar S. Pediatric Hallux Valgus. Foot Ankle Clin N Am. 2014 (19):235-243
    2. Coughlin M. Juvenile Hallux Valgus: Etiology and Treatment. Foot and Ankle International. 1995. Volume 16, Number 11. 682-697.

    Incorrect

    A bunion deformity, or hallux valgus, is a deformity of the first metatarsophalangeal joint that cause a prominence of the first metatarsal head medially. The deformity starts when the first phalanx deviates in a valgus position causing a varus deviation of the first metatarsal. The deformity is usually progressive and the big toe can ultimately start to angle toward, and cross over, the second toe. 1,2
    A bunion deformity is commonly seen in adults but can also occur in adolescents. The deformity in adults is usually caused by tight fitting shoes where the deformity in adolescents is usually caused by other factors. Pes planus, a long first metatarsal, metatarsus adductus, and positive family history are associated with developing a bunion in adolescence. Adolescent bunions generally occur in girls (88%) between the ages of 10 and 15 years old. MTP joint pain and stiffness is uncommon with adolescent bunions, unlike the adult form. Treatment of adolescent bunions includes night time bracing, shoe modification, and activity modification. A combination anatomical variants and open growth plates in adolescents contributes to a high rate of recurrence after surgical treatment, which is unlike the adult form. Surgical treatment is generally reserved for patients after skeletal maturity due to the high rate of postoperative complications in children. 1,2
    Answer C
    References
    1. Chell J, Dhar S. Pediatric Hallux Valgus. Foot Ankle Clin N Am. 2014 (19):235-243
    2. Coughlin M. Juvenile Hallux Valgus: Etiology and Treatment. Foot and Ankle International. 1995. Volume 16, Number 11. 682-697.

  14. Question 14 of 48
    14. Question

    A 13 year old male presents to your office for evaluation and management of scoliosis. He denies back or lower extremity pain. His primary care physician noticed his back asymmetry years ago but this is the first time he has addressed the issue. On the Adams forward bend test you notice left flank prominence and right rib prominence. Motor function and sensation is intact in bilateral lower extremities. AP x-ray shows a right convex Cobb angle of 32 degrees from T6-T11 and a left convex Cobb angle of 31 degrees from T11-L3. His Risser score is 4. Which choice is the most important factor in predicting curve progression?

    Correct

    Adolescent idiopathic scoliosis (AIS) is the most common type of scoliosis in patients between the ages of 10 and 18. Treatment of AIS is based on the severity of the curve and the likelihood of curve progression. Most idiopathic curves will not progress to the point where bracing or surgery is necessary. Scoliosis is defined as a lateral curvature >10 degrees. Females are ten times more likely to have curvatures of >30 degrees and are more likely to have a progressive deformity. The amount of residual skeletal growth and curve magnitude are the best predictors of whether the curve will progress or not. The Risser classification uses stages of iliac crest apophyseal ossification to grade skeletal maturity. There are six stages with stage 0 being no ossification center at the level of the iliac crest apophysis, stage 1 apophysis under 25% of iliac crest, stage 2 over 25-50%, stage 3 over 50-75%, stage 4 over 75% and stage 5 complete ossification and fusion of the iliac crest apophysis. Patients with Risser stages 0-1 with 5-19 degree curves have a 22% likelihood of curve progression while 20-29 degree curves have a 68% incidence of curve progression for the same Risser stages. Patient’s with Risser stages 2,3, and 4 with 5-19 degree curves have a 1.6% incidence of curve progression while 20-29 degree curves have an incidence of 23% curve progression for the same Riser stages. Family history and rotational prominence do not correlate with the incidence of curve progression. Thoracic curvatures are more likely to progress than lumbar curvatures. Physical exam findings of scoliosis include shoulder elevation, iliac crest height asymmetry, limb length inequality, and rib rotational deformity. 1,2
    Answer B.
    References
    1. Lonstein JE, Carlson JM. The prediction of curve progression in untreated idiopathic scoliosis during growth. JBJS 1984. 66-1067
    2. Adolescent Idiopathic Scoliosis. http://www.orthobullets.com. Accessed on 4/11/20.

    Incorrect

    Adolescent idiopathic scoliosis (AIS) is the most common type of scoliosis in patients between the ages of 10 and 18. Treatment of AIS is based on the severity of the curve and the likelihood of curve progression. Most idiopathic curves will not progress to the point where bracing or surgery is necessary. Scoliosis is defined as a lateral curvature >10 degrees. Females are ten times more likely to have curvatures of >30 degrees and are more likely to have a progressive deformity. The amount of residual skeletal growth and curve magnitude are the best predictors of whether the curve will progress or not. The Risser classification uses stages of iliac crest apophyseal ossification to grade skeletal maturity. There are six stages with stage 0 being no ossification center at the level of the iliac crest apophysis, stage 1 apophysis under 25% of iliac crest, stage 2 over 25-50%, stage 3 over 50-75%, stage 4 over 75% and stage 5 complete ossification and fusion of the iliac crest apophysis. Patients with Risser stages 0-1 with 5-19 degree curves have a 22% likelihood of curve progression while 20-29 degree curves have a 68% incidence of curve progression for the same Risser stages. Patient’s with Risser stages 2,3, and 4 with 5-19 degree curves have a 1.6% incidence of curve progression while 20-29 degree curves have an incidence of 23% curve progression for the same Riser stages. Family history and rotational prominence do not correlate with the incidence of curve progression. Thoracic curvatures are more likely to progress than lumbar curvatures. Physical exam findings of scoliosis include shoulder elevation, iliac crest height asymmetry, limb length inequality, and rib rotational deformity. 1,2
    Answer B.
    References
    1. Lonstein JE, Carlson JM. The prediction of curve progression in untreated idiopathic scoliosis during growth. JBJS 1984. 66-1067
    2. Adolescent Idiopathic Scoliosis. http://www.orthobullets.com. Accessed on 4/11/20.

  15. Question 15 of 48
    15. Question

    An 8 year-old male presents to the office with right foot pain for one month. The pain seems to be worse over the last week when his parents have noticed him starting to limp. The symptoms are made worse with running and athletic activities. Lateral radiograph of the right foot shows collapse and fragmentation of his tarsal navicular consistent with Kohler’s disease. What is the best treatment option?

    Correct

    Kohler’s disease, or osteochondrosis of the tarsal navicular, is a condition of spontaneous osteonecrosis of the navicular in pediatric patients. The condition generally occurs during the critical period of bone ossification in the foot, or during the ages of 4 to 8 years old. The navicular is the last tarsal bone to ossify and weight bearing exercise is thought to potentially disrupt the blood supply by compression from the surrounding ossified bones. Radiographs will typically show collapse of the navicular with patchy areas of sclerosis and loss of the normal trabecular bone pattern. Kohler’s disease usually resolves over time with a short period of conservative treatment. A 4-6 week period of immobilization in a walking boot or cast can promote rapid revascularization and improved outcomes. The navicular reconstitutes over several weeks and therefore athletic activities should be restricted for a few months or until complete resolution of symptoms. Pain beyond 4-5 months despite conservative treatment should raise suspicion of other diagnoses such as tumor or infection. 1,2
    Answer B.
    References
    1. Cloutier, D. Image Quiz: Osteochondrosis of the Tarsal Navicular. JBJS Journal of Orthopaedics for Physician Assistants. 6(4):e36, October-December 2018.
    2. Kohler’s disease. http://www.orthobullets.com. Accessed on 4/13/2019.

    Incorrect

    Kohler’s disease, or osteochondrosis of the tarsal navicular, is a condition of spontaneous osteonecrosis of the navicular in pediatric patients. The condition generally occurs during the critical period of bone ossification in the foot, or during the ages of 4 to 8 years old. The navicular is the last tarsal bone to ossify and weight bearing exercise is thought to potentially disrupt the blood supply by compression from the surrounding ossified bones. Radiographs will typically show collapse of the navicular with patchy areas of sclerosis and loss of the normal trabecular bone pattern. Kohler’s disease usually resolves over time with a short period of conservative treatment. A 4-6 week period of immobilization in a walking boot or cast can promote rapid revascularization and improved outcomes. The navicular reconstitutes over several weeks and therefore athletic activities should be restricted for a few months or until complete resolution of symptoms. Pain beyond 4-5 months despite conservative treatment should raise suspicion of other diagnoses such as tumor or infection. 1,2
    Answer B.
    References
    1. Cloutier, D. Image Quiz: Osteochondrosis of the Tarsal Navicular. JBJS Journal of Orthopaedics for Physician Assistants. 6(4):e36, October-December 2018.
    2. Kohler’s disease. http://www.orthobullets.com. Accessed on 4/13/2019.

  16. Question 16 of 48
    16. Question


    A 16-year-old male presents to your office with left knee pain and instability after a ski injury two days ago. He fell while skiing and the knee buckled awkwardly. He was unable to ski down the mountain after the injury. He was seen in the emergency room a few hours later where x-rays showed a tibial spine avulsion fracture (figures 1 and 2). MRI was done the next day showing an avulsion fracture of the tibial spine at the distal attachment site of the anterior cruciate ligament (ACL) with 3mm of displacement (figure 3). What is the best treatment option?

    Correct

    The tibial spine is the distal attachment site of the anterior cruciate ligament (ACL). A tibial spine fracture is uncommon but can occur before skeletal maturity as the ligament can be stronger than the attachment site leading to fracture rather than ligament rupture. Non-displaced fractures can be treated in a knee immobilizer with slight flexion (less tension on ACL) until the fracture heals. If there is any question of fracture displacement, CT can provide a precise measurement. MRI helps delineate the fracture and identify other soft tissue injury. Minimally displaced fractures may reduce with the knee placed in extension. Displaced and non-displaced fractures that can’t be reduced require surgical fixation, most commonly with an arthroscopic approach. The most common arthroscopic technique involves using suture to pull the fracture back down for an anatomic reduction. Post-operative weight bearing and range of motion is surgeon dependent, although early motion helps reduce the risk of arthrofibrosis. The expected return to sports is 3-4 months. 1,2

    Answer C.

    References

    1. Herman, Martin J. MD; Martinek, Melissa A. DO, PhD; Abzug, Joshua M. MD Complications of Tibial Eminence and Diaphyseal Fractures in Children, Journal of the American Academy of Orthopaedic Surgeons: November 2014 – Volume 22 – Issue 11 – p 730-741
    2. Osti L, Buda M, Soldati F, Del Buono A, Osti R, Maffulli N. Arthroscopic treatment of tibial eminence fracture: a systematic review of different fixation methods. Br Med Bull. 2016;118(1):73-90.
    Incorrect

    The tibial spine is the distal attachment site of the anterior cruciate ligament (ACL). A tibial spine fracture is uncommon but can occur before skeletal maturity as the ligament can be stronger than the attachment site leading to fracture rather than ligament rupture. Non-displaced fractures can be treated in a knee immobilizer with slight flexion (less tension on ACL) until the fracture heals. If there is any question of fracture displacement, CT can provide a precise measurement. MRI helps delineate the fracture and identify other soft tissue injury. Minimally displaced fractures may reduce with the knee placed in extension. Displaced and non-displaced fractures that can’t be reduced require surgical fixation, most commonly with an arthroscopic approach. The most common arthroscopic technique involves using suture to pull the fracture back down for an anatomic reduction. Post-operative weight bearing and range of motion is surgeon dependent, although early motion helps reduce the risk of arthrofibrosis. The expected return to sports is 3-4 months. 1,2

    Answer C.

    References

    1. Herman, Martin J. MD; Martinek, Melissa A. DO, PhD; Abzug, Joshua M. MD Complications of Tibial Eminence and Diaphyseal Fractures in Children, Journal of the American Academy of Orthopaedic Surgeons: November 2014 – Volume 22 – Issue 11 – p 730-741
    2. Osti L, Buda M, Soldati F, Del Buono A, Osti R, Maffulli N. Arthroscopic treatment of tibial eminence fracture: a systematic review of different fixation methods. Br Med Bull. 2016;118(1):73-90.
  17. Question 17 of 48
    17. Question


    A 12-year-old male presents to your clinic with right heel pain for 6 weeks. He noticed the pain when he started soccer practice and the pain has become progressively worse since. He is now having trouble running due to the right heal pain. On physical exam he has no obvious deformity to the foot or ankle. He has tenderness to palpation over the posterior calcaneus and mild pain with resisted plantar flexion. Lateral x-ray of the right foot is shown in figure 1. What is the most likely diagnosis?

    Correct

    Sever’s disease, or calcaneal apophysitis, is a common condition seen in pediatric and orthopedic clinics. The typical presentation includes children between the ages of 10-12 who present with heel pain after starting an athletic activity. These patients often start the sport activity with minimal to no pain but soon after participation will limp off the field or court. The etiology is unknown although the widely accepted theory is that the condition occurs during a growth spurt when the long bones grow faster in length than the tendons. Subsequently the Achilles tendon is tight creating a traction apophysitis to the calcaneal apophysis. As the foot grows in length the plantar fascia can increase in tension at its insertion site on the calcaneus, which can contribute to the pain. The diagnosis of Sever’s disease is made clinically by pain with palpation over the posterior heel (calcaneal apophysis) and open growth plates seen on radiographs. Radiographs are typically normal but may show fragmentation at the apophysis. Treatment involves activity modification, NSAIDS, and if symptoms warrant, a period of immobilization. Padded heel cups, orthotics with arch support, and heal lifts have been used without definitive evidence showing significant efficacy. 1,2

    Answer D

    References

    1. Scharfbillig RW, Jones S, Scutter SD. Sever’s disease: what does the literature really tell us?. Journal of the American Podiatric Medical Association. 2008 May;98(3):212-23.
    2. James AM, Williams CM, Haines TP. Effectiveness of interventions in reducing pain and maintaining physical activity in children and adolescents with calcaneal apophysitis (Sever’s disease): a systematic review. Journal of foot and ankle research. 2013 Dec;6(1):1-1.
    Incorrect

    Sever’s disease, or calcaneal apophysitis, is a common condition seen in pediatric and orthopedic clinics. The typical presentation includes children between the ages of 10-12 who present with heel pain after starting an athletic activity. These patients often start the sport activity with minimal to no pain but soon after participation will limp off the field or court. The etiology is unknown although the widely accepted theory is that the condition occurs during a growth spurt when the long bones grow faster in length than the tendons. Subsequently the Achilles tendon is tight creating a traction apophysitis to the calcaneal apophysis. As the foot grows in length the plantar fascia can increase in tension at its insertion site on the calcaneus, which can contribute to the pain. The diagnosis of Sever’s disease is made clinically by pain with palpation over the posterior heel (calcaneal apophysis) and open growth plates seen on radiographs. Radiographs are typically normal but may show fragmentation at the apophysis. Treatment involves activity modification, NSAIDS, and if symptoms warrant, a period of immobilization. Padded heel cups, orthotics with arch support, and heal lifts have been used without definitive evidence showing significant efficacy. 1,2

    Answer D

    References

    1. Scharfbillig RW, Jones S, Scutter SD. Sever’s disease: what does the literature really tell us?. Journal of the American Podiatric Medical Association. 2008 May;98(3):212-23.
    2. James AM, Williams CM, Haines TP. Effectiveness of interventions in reducing pain and maintaining physical activity in children and adolescents with calcaneal apophysitis (Sever’s disease): a systematic review. Journal of foot and ankle research. 2013 Dec;6(1):1-1.
  18. Question 18 of 48
    18. Question


    A 12-year-old female presents to the emergency room with severe left ankle pain after twisting her ankle and falling while running earlier in the day. On physical exam of the left ankle her skin is intact with mild swelling over the medial and lateral aspects of the ankle. Ankle range of motion is limited due to pain and swelling. AP and lateral radiographs are shown in figures 1 and 2. What is the next best step in treatment?

    Correct

    A triplane fracture is a unique pattern that occurs at the distal tibial epiphysis during the period of physeal closure or around 12-15 years of age. Triplane refers to the pattern of fracture in the sagittal, transverse, and coronal planes. Specifically, the pattern includes a vertical fracture through the epiphysis, a horizontal fracture through the physis, and an oblique fracture through the metaphysis. The presence of a metaphyseal fractures distinguishes a triplane fracture from a Tillaux fracture. Triplane fractures most commonly occur from a twisting injury during athletic participation. The fracture can be in multiple fragments which is difficult to visualize on plain radiographs. Like most displaced intra-articular distal tibia fractures, CT is necessary to better visualize the fracture pattern and determine fracture displacement. Indications for surgery for triplane fractures include >2mm of intra-articular displacement. Without an anatomical reduction patients are at risk for premature physeal closure, ankle deformity, and osteoarthritis. 1,2

    Answer A.

    References

    1. Schnetzler, Kent A. MD, MS; Hoernschemeyer, Daniel MD The Pediatric Triplane Ankle Fracture, Journal of the American Academy of Orthopaedic Surgeons: December 2007 – Volume 15 – Issue 12 – p 738-747
    2. Liporace FA, Yoon RS, Kubiak EN, Parisi DM, Koval KJ, Feldman DS, Egol KA. Does adding computed tomography change the diagnosis and treatment of Tillaux and triplane pediatric ankle fractures?. Orthopedics. 2012 Feb 17;35(2):e208-12.
    Incorrect

    A triplane fracture is a unique pattern that occurs at the distal tibial epiphysis during the period of physeal closure or around 12-15 years of age. Triplane refers to the pattern of fracture in the sagittal, transverse, and coronal planes. Specifically, the pattern includes a vertical fracture through the epiphysis, a horizontal fracture through the physis, and an oblique fracture through the metaphysis. The presence of a metaphyseal fractures distinguishes a triplane fracture from a Tillaux fracture. Triplane fractures most commonly occur from a twisting injury during athletic participation. The fracture can be in multiple fragments which is difficult to visualize on plain radiographs. Like most displaced intra-articular distal tibia fractures, CT is necessary to better visualize the fracture pattern and determine fracture displacement. Indications for surgery for triplane fractures include >2mm of intra-articular displacement. Without an anatomical reduction patients are at risk for premature physeal closure, ankle deformity, and osteoarthritis. 1,2

    Answer A.

    References

    1. Schnetzler, Kent A. MD, MS; Hoernschemeyer, Daniel MD The Pediatric Triplane Ankle Fracture, Journal of the American Academy of Orthopaedic Surgeons: December 2007 – Volume 15 – Issue 12 – p 738-747
    2. Liporace FA, Yoon RS, Kubiak EN, Parisi DM, Koval KJ, Feldman DS, Egol KA. Does adding computed tomography change the diagnosis and treatment of Tillaux and triplane pediatric ankle fractures?. Orthopedics. 2012 Feb 17;35(2):e208-12.
  19. Question 19 of 48
    19. Question


    An 11-year-old female presents to your office with her parents complaining of severe right hip pain and difficulty walking. She denies having a known injury and the pain has progressively increased over the last 4 weeks. She is very healthy and active with no underlying medical conditions. On physical exam she has pain with hip motion but no obvious skin changes or deformity. AP pelvis and lateral x-rays are shown in figures 1 and 2, respectively. What is the next best step in treatment?

    Correct

    The patient is presenting with a chronic slipped capital femoral epiphysis (SCFE). Chronic SCFE is defined as symptoms lasting >3 weeks and is by far the most common form, accounting for 85% of all patients who present with the condition. The stability of SCFE is based on the patient’s ability to bear weight. If patients can’t put any weight on the leg the SCFE is unstable whereas the ability to weight bear represents a stable pattern. Given the patient was able to walk in, although with pain, her SCFE would be classified as stable. The patient’s x-rays show a grade I mild slip which appears chronic. Given the duration of symptoms and the patients difficulty with bearing weight, the SCFE is still not healed. MRI has a role in diagnosing early pre-slip SCFE in the presence of normal radiographs, however this patient has growth plate changes noted. The treatment of choice for stable and unstable SCFE is single or double screw fixation to prevent further slippage of the growth plate. Unstable and severe slips may require open reduction of the growth plate with fixation. 1,2

    Answer D.

    References

    1. Aronsson, David D. MD; Loder, Randall T. MD; Breur, Gert J. DVM, PhD; Weinstein, Stuart L. MD Slipped Capital Femoral Epiphysis: Current Concepts, Journal of the American Academy of Orthopaedic Surgeons: November 2006 – Volume 14 – Issue 12 – p 666-679
    2. Wylie JD, Novais EN. Evolving Understanding of and Treatment Approaches to Slipped Capital Femoral Epiphysis. Curr Rev Musculoskelet Med. 2019;12(2):213-219. doi:10.1007/s12178-019-09547-5
    Incorrect

    The patient is presenting with a chronic slipped capital femoral epiphysis (SCFE). Chronic SCFE is defined as symptoms lasting >3 weeks and is by far the most common form, accounting for 85% of all patients who present with the condition. The stability of SCFE is based on the patient’s ability to bear weight. If patients can’t put any weight on the leg the SCFE is unstable whereas the ability to weight bear represents a stable pattern. Given the patient was able to walk in, although with pain, her SCFE would be classified as stable. The patient’s x-rays show a grade I mild slip which appears chronic. Given the duration of symptoms and the patients difficulty with bearing weight, the SCFE is still not healed. MRI has a role in diagnosing early pre-slip SCFE in the presence of normal radiographs, however this patient has growth plate changes noted. The treatment of choice for stable and unstable SCFE is single or double screw fixation to prevent further slippage of the growth plate. Unstable and severe slips may require open reduction of the growth plate with fixation. 1,2

    Answer D.

    References

    1. Aronsson, David D. MD; Loder, Randall T. MD; Breur, Gert J. DVM, PhD; Weinstein, Stuart L. MD Slipped Capital Femoral Epiphysis: Current Concepts, Journal of the American Academy of Orthopaedic Surgeons: November 2006 – Volume 14 – Issue 12 – p 666-679
    2. Wylie JD, Novais EN. Evolving Understanding of and Treatment Approaches to Slipped Capital Femoral Epiphysis. Curr Rev Musculoskelet Med. 2019;12(2):213-219. doi:10.1007/s12178-019-09547-5
  20. Question 20 of 48
    20. Question


    An 11-year-old girl presents to your office with left knee pain for 2 months. The pain seems to be located at the front of the knee just past the patella on the lower leg. The pain is made worse with running and jumping. She has tried ice and anti-inflammatories which offered mild relief. On physical exam the patient has no deformity or knee effusion. She has tenderness to palpation over the tibial tubercle on the left knee. AP and lateral x-rays are shown in figures 1 and 2. What is the next best step in treatment?

    Correct

    Osgood-Schlatter disease (OSD) is a common knee condition seen in active adolescents which results from traction apophysitis at the tibial tubercle. Symptoms start during a period of rapid skeletal growth while patients participate in athletic activities. The diagnosis is straightforward with pain to palpation over the tibial tubercle and pain that is made worse with running and jumping activities. Radiographs are usually normal but may show irregularity and fragmentation of the tibial tubercle. Advanced imaging with MRI and/or CT is not indicated to confirm the diagnosis. Treatment involves NSAIDS, ice, activity modification and quadriceps stretching. Patients may have to pause athletic activities for a short period if pain is significant, however most patients can play through mild discomfort. OSD can be a nagging type of condition that lasts up to 12-18 months but generally resolves when patients reach skeletal maturity. Maintaining quadriceps flexibility through sport activities is critical to managing symptoms. 1,2

    Answer A.

     

    1. Bloom OJ, Mackler L. What is the best treatment for Osgood-Schlatter disease?. Clinical Inquiries, 2004 (MU). 2004.
    2. Circi E, Atalay Y, Beyzadeoglu T. Treatment of Osgood–Schlatter disease: review of the literature. Musculoskeletal surgery. 2017 Dec;101(3):195-200.
    Incorrect

    Osgood-Schlatter disease (OSD) is a common knee condition seen in active adolescents which results from traction apophysitis at the tibial tubercle. Symptoms start during a period of rapid skeletal growth while patients participate in athletic activities. The diagnosis is straightforward with pain to palpation over the tibial tubercle and pain that is made worse with running and jumping activities. Radiographs are usually normal but may show irregularity and fragmentation of the tibial tubercle. Advanced imaging with MRI and/or CT is not indicated to confirm the diagnosis. Treatment involves NSAIDS, ice, activity modification and quadriceps stretching. Patients may have to pause athletic activities for a short period if pain is significant, however most patients can play through mild discomfort. OSD can be a nagging type of condition that lasts up to 12-18 months but generally resolves when patients reach skeletal maturity. Maintaining quadriceps flexibility through sport activities is critical to managing symptoms. 1,2

    Answer A.

     

    1. Bloom OJ, Mackler L. What is the best treatment for Osgood-Schlatter disease?. Clinical Inquiries, 2004 (MU). 2004.
    2. Circi E, Atalay Y, Beyzadeoglu T. Treatment of Osgood–Schlatter disease: review of the literature. Musculoskeletal surgery. 2017 Dec;101(3):195-200.
  21. Question 21 of 48
    21. Question


    A 14-year-old boy presents to the office accompanied by his parents with complaints of right lateral elbow pain for 6 months. He denies a known injury and the pain is getting worse over the last month since baseball season started. He is involved in gymnastics and baseball most months throughout the year. On physical exam the patient has tenderness to palpation over the radiocapitellar joint. Range of motion of the elbow is full and equal to the contralateral side. AP x-ray of the right elbow (figure 1) shows a focal radiolucent lesion of the capitellum with flattening of the articular surface. MRI of the elbow in figure 2 shows a cyst in the capitellum with a joint effusion and loose bodies. What is the best treatment option

    Correct

    Osteochondritis dissecans (OCD) most commonly occurs at the capitellum in adolescent athletes. Gymnastics and baseball have the highest incidence of OCD of the capitellum which is likely attributed to compression of the radiocapitellar joint during hand stands and overhead throwing. This patient is presenting with advanced OCD which is indicated by radiographic changes and the presence of loose bodies in the elbow joint. Unstable OCD lesions with loose bodies generally require surgery. An arthroscopic or open excision of the loose bodies must be performed to prevent mechanical symptoms and recurrent effusions. Fixation of unstable lesions or osteochondral fragment removal can be performed depending on the size and stability of the lesion. In general fragments consisting of 50% or more of the capitellum require fixation. Osteochondral autograft plugs are used for large defects to restore the articular surface. 1,2

    Answer C.

    References

    1. Baker III, Champ L., Anthony A. Romeo, and Champ L. Baker Jr. “Osteochondritis dissecans of the capitellum.” The American journal of sports medicine 38.9 (2010): 1917-1928.
    2. Ruchelsman DE, Hall MP, Youm T. Osteochondritis Dissecans of the Capitellum: Current Concepts. J Am Acad Orthop Surg 2010;18:

     

    Incorrect

    Osteochondritis dissecans (OCD) most commonly occurs at the capitellum in adolescent athletes. Gymnastics and baseball have the highest incidence of OCD of the capitellum which is likely attributed to compression of the radiocapitellar joint during hand stands and overhead throwing. This patient is presenting with advanced OCD which is indicated by radiographic changes and the presence of loose bodies in the elbow joint. Unstable OCD lesions with loose bodies generally require surgery. An arthroscopic or open excision of the loose bodies must be performed to prevent mechanical symptoms and recurrent effusions. Fixation of unstable lesions or osteochondral fragment removal can be performed depending on the size and stability of the lesion. In general fragments consisting of 50% or more of the capitellum require fixation. Osteochondral autograft plugs are used for large defects to restore the articular surface. 1,2

    Answer C.

    References

    1. Baker III, Champ L., Anthony A. Romeo, and Champ L. Baker Jr. “Osteochondritis dissecans of the capitellum.” The American journal of sports medicine 38.9 (2010): 1917-1928.
    2. Ruchelsman DE, Hall MP, Youm T. Osteochondritis Dissecans of the Capitellum: Current Concepts. J Am Acad Orthop Surg 2010;18:

     

  22. Question 22 of 48
    22. Question


    A 5-year-old girl presents to an urgent care with left elbow pain after falling off the monkey bars a few hours earlier. She landed on the left hand with the arm in extension. She has been unable to move the elbow since the injury. AP and lateral x-rays of the left elbow are shown in figures 1 and 2, respectively. What is the best treatment option for this patient?

    Correct

    A Monteggia fracture is characterized by a proximal third ulnar fracture with a dislocation of the radial head. These fractures are rare in children and can be missed at the initial presentation in up to 50% of pediatric patients.  A missed diagnosis can cause disabling consequence as a persistent radial head dislocation can start to cause permanent loss of elbow motion and stability after just two weeks from injury. When establishing a diagnosis, the patients range of motion should be carefully scrutinized for full elbow flexion and extension and full forearm supination and pronation. It is crucial for providers to be able to recognize an intact radiocapitellar joint on radiographs. This can often be difficult with the numerous ossification centers of the elbow. If there is any confusion on radiographic interpretation an x-ray of the contralateral elbow should be obtained for a side to side comparison. The treatment of choice is an acute closed reduction and long arm cast with the forearm placed in supination. The radial head will often reduce spontaneously when ulna length is restored. Closed reduction should be performed as soon as possible as the reduction becomes more difficult as time from injury passes. Chronic Monteggia fractures, defined as a dislocation beyond 4 weeks, usually require an open reduction. Open reduction and internal fixation is only performed in patients with closed growth plates. 1, 2

    Answer B.

    References

    1. Hubbard James, MD; Chauhan Aakash, MD, MBA; Fitzgerald Ryan, MD; Abrams Reid, MD; Mubarak Scott, MD; Sangimino Mark, MD;. Missed Pediatric Monteggia Fractures. JBJS Reviews – Review Article: 05 June 2018 – Volume 6 – Issue 6 – p. e2-e2
    2. Monteggia fracture. http://www.orthobullets.com. Accessed 9/23/2020
    Incorrect

    A Monteggia fracture is characterized by a proximal third ulnar fracture with a dislocation of the radial head. These fractures are rare in children and can be missed at the initial presentation in up to 50% of pediatric patients.  A missed diagnosis can cause disabling consequence as a persistent radial head dislocation can start to cause permanent loss of elbow motion and stability after just two weeks from injury. When establishing a diagnosis, the patients range of motion should be carefully scrutinized for full elbow flexion and extension and full forearm supination and pronation. It is crucial for providers to be able to recognize an intact radiocapitellar joint on radiographs. This can often be difficult with the numerous ossification centers of the elbow. If there is any confusion on radiographic interpretation an x-ray of the contralateral elbow should be obtained for a side to side comparison. The treatment of choice is an acute closed reduction and long arm cast with the forearm placed in supination. The radial head will often reduce spontaneously when ulna length is restored. Closed reduction should be performed as soon as possible as the reduction becomes more difficult as time from injury passes. Chronic Monteggia fractures, defined as a dislocation beyond 4 weeks, usually require an open reduction. Open reduction and internal fixation is only performed in patients with closed growth plates. 1, 2

    Answer B.

    References

    1. Hubbard James, MD; Chauhan Aakash, MD, MBA; Fitzgerald Ryan, MD; Abrams Reid, MD; Mubarak Scott, MD; Sangimino Mark, MD;. Missed Pediatric Monteggia Fractures. JBJS Reviews – Review Article: 05 June 2018 – Volume 6 – Issue 6 – p. e2-e2
    2. Monteggia fracture. http://www.orthobullets.com. Accessed 9/23/2020
  23. Question 23 of 48
    23. Question


    A 18-year-old male presents to your office with a growth on his right knee for a few years. The mass has progressively enlarged over the last 2-3 years and is now causing catching and locking as he bends the knee. He denies night pain and unexplained weight loss. On physical exam of the right knee there is a palpable hard mass to the anterolateral aspect of the knee adjacent to the superior portion of the patella and the patella is sitting slightly more medial than the left knee. The mass is non-tender to palpation and non-mobile. Figures 1 and 2 show a broad based tumor emanating from the distal metaphysis of the right femur. Axial MRI (figure 3) shows a dome shaped osseous lesion emanating from the lateral aspect of the distal femur metaphysis. There is a slender cartilaginous cap measuring 3 mm thick. What is the best treatment option for this patient?

    Correct

    The patient is presenting with a benign osteochondroma. This osteochondroma is uncommon in that it is located on the anterolateral femur causing mechanical impingement on the patella during knee motion. An osteochondroma is the most common benign tumor of childhood and young adults. Solitary osteochondromas may, although very rare (<1%), transform to a malignant chondrosarcoma over time. It is essential to identify tumor characteristics on radiographs and MRI to establish the correct diagnosis. Radiographic features of a benign lesion include tumor location at the metaphysis, the cortex of the lesion continuous with the cortex of the surrounding femur, and no metastases. Patterns of bone destruction signal a more malignant nature of a tumor. These patterns may include moth-eaten bone, a fast growing tumor, and an absent sclerotic boarder. MRI is the study of choice to assess the characteristics of the lesion to help determine if the lesion is benign or malignant. A cartilaginous cap greater than 1.5 cm in thickness is suspicious for malignant transformation. Surgical treatment of a solitary osteochondroma involves a marginal resection that includes the base of the stalk, the cartilage cap, and overlying periosteum. A wide resection is the treatment of choice for a chondrosarcoma. 1,2,3

    Answer C.

    References

    1. August Image Quiz: Osteochondroma. Journal of Orthopaedics for Physician Assistants – Image Quiz: 01 August 2015 – Volume 3 – Issue 3 – p. 16
    2. Aboulafia, Albert J. MD; Kennon, Robert E. MD; Jelinek, James S. MD Benign Bone Tumors of Childhood, Journal of the American Academy of Orthopaedic Surgeons: November 1999 – Volume 7 – Issue 6 – p 377-388.
    3. Osteochondroma. http://www.radiopaedia.org. Accessed on 3/28/21.
    Incorrect

    The patient is presenting with a benign osteochondroma. This osteochondroma is uncommon in that it is located on the anterolateral femur causing mechanical impingement on the patella during knee motion. An osteochondroma is the most common benign tumor of childhood and young adults. Solitary osteochondromas may, although very rare (<1%), transform to a malignant chondrosarcoma over time. It is essential to identify tumor characteristics on radiographs and MRI to establish the correct diagnosis. Radiographic features of a benign lesion include tumor location at the metaphysis, the cortex of the lesion continuous with the cortex of the surrounding femur, and no metastases. Patterns of bone destruction signal a more malignant nature of a tumor. These patterns may include moth-eaten bone, a fast growing tumor, and an absent sclerotic boarder. MRI is the study of choice to assess the characteristics of the lesion to help determine if the lesion is benign or malignant. A cartilaginous cap greater than 1.5 cm in thickness is suspicious for malignant transformation. Surgical treatment of a solitary osteochondroma involves a marginal resection that includes the base of the stalk, the cartilage cap, and overlying periosteum. A wide resection is the treatment of choice for a chondrosarcoma. 1,2,3

    Answer C.

    References

    1. August Image Quiz: Osteochondroma. Journal of Orthopaedics for Physician Assistants – Image Quiz: 01 August 2015 – Volume 3 – Issue 3 – p. 16
    2. Aboulafia, Albert J. MD; Kennon, Robert E. MD; Jelinek, James S. MD Benign Bone Tumors of Childhood, Journal of the American Academy of Orthopaedic Surgeons: November 1999 – Volume 7 – Issue 6 – p 377-388.
    3. Osteochondroma. http://www.radiopaedia.org. Accessed on 3/28/21.
  24. Question 24 of 48
    24. Question


    An 11-year-old male presents to your clinic with left ankle pain after twisting his ankle while playing basketball 2 days ago. He has had pain and swelling in the ankle since and is having trouble bearing weight. On physical exam there is mild bruising and swelling to the lateral ankle. He has tenderness to palpation over the distal fibula and anterior talofibular ligament (ATFL). AP and lateral radiographs are shown in figures 1 and 2. Which statement is true regarding the possibility of a lateral ankle sprain vs. a Salter-Harris I distal fibular fracture in this patient?

     

    Correct

    Lateral ankle injuries are commonly seen in the pediatric population. In fact, ankle trauma is the most common pediatric injury. In skeletally immature patients it can be hard to conclude whether the injury is a lateral ankle sprain or a Salter-Harris I (non-displaced) fracture. A careful physical exam is critical in making the diagnosis, however kids often have difficulty pinpointing the exact location of the pain. Historically most lateral ankle injuries have been presumed to be growth plate injuries. The theory is that in growing kids, the physis is weaker than the surrounding ligaments and therefore injury to the growth plate is more likely. This theory has since been proven untrue. Boutis et al. performed an MRI on 135 pediatric patients who sustained a lateral ankle injury and found that almost all patients injured the lateral ligaments and not the physis. Only 4 of 135 (3%) patients had an MRI confirmed Salter-Harris I fracture. Salter-Harris I fractures and lateral ankle sprains have a similar treatment course which includes short-term immobilization or bracing (usually 3 weeks depending on symptoms) with a gradual return to activities as tolerated. Physical therapy is an important treatment for lateral ankle sprains to regain ankle motion, reduce swelling, and strengthening. Misdiagnosing lateral ankle sprains for physeal injuries often prevents physical therapy referral and leads to unnecessary immobilization and follow-up radiographs.

    Answer D.

    References

    1. Boutis, Kathy, et al. “Radiograph-negative lateral ankle injuries in children: occult growth plate fracture or sprain?.” JAMA pediatrics 170.1 (2016): e154114-e154114.
    2. Rougereau G, Noailles T, El Khoury G, Bauer T, Langlais T, Hardy A. Is lateral ankle sprain of the child and adolescent a myth or a reality? A systematic review of the literature. Foot and Ankle Surgery. 2021 May 1.
    Incorrect

    Lateral ankle injuries are commonly seen in the pediatric population. In fact, ankle trauma is the most common pediatric injury. In skeletally immature patients it can be hard to conclude whether the injury is a lateral ankle sprain or a Salter-Harris I (non-displaced) fracture. A careful physical exam is critical in making the diagnosis, however kids often have difficulty pinpointing the exact location of the pain. Historically most lateral ankle injuries have been presumed to be growth plate injuries. The theory is that in growing kids, the physis is weaker than the surrounding ligaments and therefore injury to the growth plate is more likely. This theory has since been proven untrue. Boutis et al. performed an MRI on 135 pediatric patients who sustained a lateral ankle injury and found that almost all patients injured the lateral ligaments and not the physis. Only 4 of 135 (3%) patients had an MRI confirmed Salter-Harris I fracture. Salter-Harris I fractures and lateral ankle sprains have a similar treatment course which includes short-term immobilization or bracing (usually 3 weeks depending on symptoms) with a gradual return to activities as tolerated. Physical therapy is an important treatment for lateral ankle sprains to regain ankle motion, reduce swelling, and strengthening. Misdiagnosing lateral ankle sprains for physeal injuries often prevents physical therapy referral and leads to unnecessary immobilization and follow-up radiographs.

    Answer D.

    References

    1. Boutis, Kathy, et al. “Radiograph-negative lateral ankle injuries in children: occult growth plate fracture or sprain?.” JAMA pediatrics 170.1 (2016): e154114-e154114.
    2. Rougereau G, Noailles T, El Khoury G, Bauer T, Langlais T, Hardy A. Is lateral ankle sprain of the child and adolescent a myth or a reality? A systematic review of the literature. Foot and Ankle Surgery. 2021 May 1.
  25. Question 25 of 48
    25. Question


    A 2 year old boy is brought to your office by his mother with a 3 day history of limping and left lower extremity pain. The child was at daycare when one staff member noticed the child limping on the playground. On physical exam you notice mild swelling to the distal leg but otherwise no deformity or bruising. The child refuses to ambulate on the left leg or let you examine him. AP and lateral x-rays are shown in figures 1 and 2. You suspect a toddler’s fracture. Which statement is true regarding a toddler’s fracture?

    Correct

    A toddler’s fracture is a low impact fracture of the distal one-third tibia with an intact fibula that usually occurs in children under the age of 2.5 years. A toddler’s fracture can occur as the tibia is rotated and the foot is fixed to the ground which causes the classic spiral fracture pattern. The injury can occur while running or falling from a height. The injury frequently goes unnoticed by parents or caregivers which may delay the presentation to clinic. Common physical exam findings include swelling and refusal to bear weight on the affected extremity. The fracture is generally non-displaced and may not be evident on initial radiographs. A toddler that presents to the office with a limp, pain along the affected tibia, and normal x-rays should be treated as a fracture. Treatment involves placing the child in a long leg cast with the knee flexed at 30 degrees to avoid weight bearing. Healing callus may be evident along the tibial periosteum as early as 2-3 weeks after the injury and may be the only evidence that a fracture occurred. The differential diagnosis in a limping toddler may include other subtle fractures that are caused by minor trauma including metatarsal buckle fractures, fractures of the midfoot, and fractures of the proximal tibia and distal tibia and fibula. 1,2

    Answer C

    References

    1. John, Susan D., Chetan S. Moorthy, and Leonard E. Swischuk. “Expanding the concept of the toddler’s fracture.” Radiographics 17.2 (1997): 367-376.
    2. Halsey, Matthew F., et al. “Toddler’s fracture: presumptive diagnosis and treatment.” Journal of Pediatric Orthopaedics 21.2 (2001): 152-156.
    Incorrect

    A toddler’s fracture is a low impact fracture of the distal one-third tibia with an intact fibula that usually occurs in children under the age of 2.5 years. A toddler’s fracture can occur as the tibia is rotated and the foot is fixed to the ground which causes the classic spiral fracture pattern. The injury can occur while running or falling from a height. The injury frequently goes unnoticed by parents or caregivers which may delay the presentation to clinic. Common physical exam findings include swelling and refusal to bear weight on the affected extremity. The fracture is generally non-displaced and may not be evident on initial radiographs. A toddler that presents to the office with a limp, pain along the affected tibia, and normal x-rays should be treated as a fracture. Treatment involves placing the child in a long leg cast with the knee flexed at 30 degrees to avoid weight bearing. Healing callus may be evident along the tibial periosteum as early as 2-3 weeks after the injury and may be the only evidence that a fracture occurred. The differential diagnosis in a limping toddler may include other subtle fractures that are caused by minor trauma including metatarsal buckle fractures, fractures of the midfoot, and fractures of the proximal tibia and distal tibia and fibula. 1,2

    Answer C

    References

    1. John, Susan D., Chetan S. Moorthy, and Leonard E. Swischuk. “Expanding the concept of the toddler’s fracture.” Radiographics 17.2 (1997): 367-376.
    2. Halsey, Matthew F., et al. “Toddler’s fracture: presumptive diagnosis and treatment.” Journal of Pediatric Orthopaedics 21.2 (2001): 152-156.
  26. Question 26 of 48
    26. Question

    A 5 year old male presents to your office with right hip pain and limp for the past month. His mother has noticed his symptoms and is concerned as the child did not sustain any obvious injury. The mom denies any recent viral infection, fever, chills, or sweats. He seems to be acting fine other than the limp and occasional complaints of hip pain. On physical exam he has a mild decrease in hip abduction and internal rotation on the right compared to the left. He has no leg length discrepancy. AP and frog leg lateral x-rays of the hip are shown in figures 1 and 2. What is the best treatment option for this patient?

    Correct

    The child is presenting with hip pain, a limp, and collapse of the femoral head on x-rays which is consistent with the diagnosis of Legg-Calve-Perthes Disease (LCPD). LCPD is an idiopathic avascular necrosis of the femoral head that generally occurs between the ages of 5 and 8 years old. The most common presenting symptoms are mild hip pain and a limp. The natural course of the condition starts with sclerosis of the epiphysis, loss of epiphyseal height, fragmentation and fissuring of the epiphysis, followed by maximal collapse and fissuring, and finally new bone formation and healing. The course of symptom onset to resolution can last over a period of 2-4 years. AP and frog leg lateral x-rays can be performed every 3-4 months to ensure the femoral head is contained in the acetabulum and to track the course of the disease. The amount of femoral head collapse correlates with outcome, including the likelihood of developing a hip deformity and secondary osteoarthritis. Age of symptom onset is a significant predictor of outcome as younger patients have more time to grow out of any residual deformities. Age of onset <6 years of age can be treated successfully with nonsurgical management (activity modification based on symptoms). Bracing and physical therapy are not necessary or helpful for improving outcomes in patients <6 without severe epiphyseal collapse. Age of onset between 6 and 8 years of age offers a less predictable course and surgical vs. nonsurgical outcomes are comparable in this group. Patients with symptom onset at 8 years of age and older are more prone to femoral head and acetabular abnormalities and poor hip function. Surgery is often performed early in the course of the disease for these patients to minimize the risk of secondary degenerative arthritis. 1,2
    Answer A.
    References
    1. Herring JA, Kim HT, Browne R. Legg-Calvé-Perthes disease: part II: prospective multicenter study of the effect of treatment on outcome. JBJS. 2004 Oct 1;86(10):2121-34.
    2. Kim, Harry K. W. MD. Legg-Calvé-Perthes Disease. American Academy of Orthopaedic Surgeon: November 2010 – Volume 18 – Issue 11 – p 676-686

    Incorrect

    The child is presenting with hip pain, a limp, and collapse of the femoral head on x-rays which is consistent with the diagnosis of Legg-Calve-Perthes Disease (LCPD). LCPD is an idiopathic avascular necrosis of the femoral head that generally occurs between the ages of 5 and 8 years old. The most common presenting symptoms are mild hip pain and a limp. The natural course of the condition starts with sclerosis of the epiphysis, loss of epiphyseal height, fragmentation and fissuring of the epiphysis, followed by maximal collapse and fissuring, and finally new bone formation and healing. The course of symptom onset to resolution can last over a period of 2-4 years. AP and frog leg lateral x-rays can be performed every 3-4 months to ensure the femoral head is contained in the acetabulum and to track the course of the disease. The amount of femoral head collapse correlates with outcome, including the likelihood of developing a hip deformity and secondary osteoarthritis. Age of symptom onset is a significant predictor of outcome as younger patients have more time to grow out of any residual deformities. Age of onset <6 years of age can be treated successfully with nonsurgical management (activity modification based on symptoms). Bracing and physical therapy are not necessary or helpful for improving outcomes in patients <6 without severe epiphyseal collapse. Age of onset between 6 and 8 years of age offers a less predictable course and surgical vs. nonsurgical outcomes are comparable in this group. Patients with symptom onset at 8 years of age and older are more prone to femoral head and acetabular abnormalities and poor hip function. Surgery is often performed early in the course of the disease for these patients to minimize the risk of secondary degenerative arthritis. 1,2
    Answer A.
    References
    1. Herring JA, Kim HT, Browne R. Legg-Calvé-Perthes disease: part II: prospective multicenter study of the effect of treatment on outcome. JBJS. 2004 Oct 1;86(10):2121-34.
    2. Kim, Harry K. W. MD. Legg-Calvé-Perthes Disease. American Academy of Orthopaedic Surgeon: November 2010 – Volume 18 – Issue 11 – p 676-686

  27. Question 27 of 48
    27. Question

    A 17-year-old female presents to your office with left foot pain for the last 8 weeks. She is an avid runner and 6 weeks ago had an x-ray showing a subtle periosteal reaction around the distal third metatarsal (figure 1). She was placed in a walking boot and has avoided running over the last 6 weeks. AP x-ray (figure 2) taken in the office show today significant callus formation around the distal third metatarsal. On physical exam she had mild tenderness to palpation over the distal third metatarsal. The patient is concerned about how the stress fracture occurred. Which statement is true regarding the risk of a stress fracture?

    Correct

    The incidence of lower extremity stress fractures is much more common in female athletes compared to males. Metatarsal stress fractures are particularly common among female runners and dancers. The second and third metatarsal are the most common metatarsals to have a stress fracture due to their length and relative immobility. Early radiographic findings for a stress fracture may include a subtle radiolucency of the shaft and/or cortex. Subtle periosteal bone formation may be seen as early as 2 weeks from injury and thicker callus formation is usually seen at 4-6 weeks post-injury. Risk factors for stress fractures include inadequate dietary intake (which is associated with athletic amenorrhea), lack of rest to allow for proper bone turnover and healing, and a serum 25(OH)D level <40 ng/mL. Treatment involves resting the foot with an immobilizer weight bearing as tolerated. With a hard soled shoe taking pressure off the metatarsals, non-weight bearing restrictions is unnecessary. 1,2
    Answer C.
    References
    1. Frank RM, Romeo AA, Bush-Joseph CA, Bach BR. Injuries to the Female Athlete in 2017. Part I: General Considerations, Concussions, Stress Fractures, and the Female Athlete Triad. 2017 Oct; JBJS Rev, 5(10):e4
    2. Miller JR, Dunn KW, Ciliberti Jr LJ, Patel RD, Swanson BA. Association of vitamin D with stress fractures: a retrospective cohort study. The Journal of Foot and Ankle Surgery. 2016 Jan 1;55(1):117-20.

    Incorrect

    The incidence of lower extremity stress fractures is much more common in female athletes compared to males. Metatarsal stress fractures are particularly common among female runners and dancers. The second and third metatarsal are the most common metatarsals to have a stress fracture due to their length and relative immobility. Early radiographic findings for a stress fracture may include a subtle radiolucency of the shaft and/or cortex. Subtle periosteal bone formation may be seen as early as 2 weeks from injury and thicker callus formation is usually seen at 4-6 weeks post-injury. Risk factors for stress fractures include inadequate dietary intake (which is associated with athletic amenorrhea), lack of rest to allow for proper bone turnover and healing, and a serum 25(OH)D level <40 ng/mL. Treatment involves resting the foot with an immobilizer weight bearing as tolerated. With a hard soled shoe taking pressure off the metatarsals, non-weight bearing restrictions is unnecessary. 1,2
    Answer C.
    References
    1. Frank RM, Romeo AA, Bush-Joseph CA, Bach BR. Injuries to the Female Athlete in 2017. Part I: General Considerations, Concussions, Stress Fractures, and the Female Athlete Triad. 2017 Oct; JBJS Rev, 5(10):e4
    2. Miller JR, Dunn KW, Ciliberti Jr LJ, Patel RD, Swanson BA. Association of vitamin D with stress fractures: a retrospective cohort study. The Journal of Foot and Ankle Surgery. 2016 Jan 1;55(1):117-20.

  28. Question 28 of 48
    28. Question

    A 13 year old male presents to the office with significant left ankle pain after a twisting injury while playing basketball earlier in the day. He planted to make a quick move and felt the ankle give out. On physical exam he has significant pain to palpation over the medial malleolus with mild swelling to the medial ankle. AP and lateral x-rays of the left ankle are shown in figures 1 and 2. The fracture line appears to start at the medial malleolus and a subtle fracture line extends to the epiphysis. Which choice below best describes this fracture?

    Correct

    Ankle fractures are a very common pediatric injury, only behind distal radius and finger injuries as the most common fracture locations involving growth plates. The distal tibial physis contributes to 40% growth of the tibia so an ankle injury that causes growth arrest can be devastating. Distal tibial physis closure generally occurs by 16 years of age for boys and 14 for girls. Before this time the surrounding ligament structures can be stronger than the growth plates, which leads to a tendency for growth plate fractures over ankle sprains in this age group. Over a period of 18 months, the distal tibial physis closes first in the middle of the physis, then the medial part, and lastly the lateral side. A Tillaux fracture is a Salter-Harris III fracture of the anterolateral portion of the distal tibia, which results from an epiphyseal avulsion at the attachment site of the anterior talofibular ligament. Triplane fractures have fracture lines in 3 planes which involve the metaphyseal fragment posteriorly and an epiphyseal fragment which is more often lateral than medial. The patients fracture appears to involve the epiphysis and physis which is characteristic of a Salter-Harris Type III fracture. 1,2

     

    Answer D

    References

    1. Kay, Robert M. MD; Matthys, Gary A. MD. Pediatric Ankle Fractures: Evaluation and Treatment. Journal of the American Academy of Orthopaedic Surgeons 9(4):p 268-278, July 2001.
    2. Blackburn EW, Aronsson D, Rubright J, Lisle J. Current Concepts Review: Ankle fractures in Children. J Bone Joint Surg Am. 2012;94:1234-44
    Incorrect

    Ankle fractures are a very common pediatric injury, only behind distal radius and finger injuries as the most common fracture locations involving growth plates. The distal tibial physis contributes to 40% growth of the tibia so an ankle injury that causes growth arrest can be devastating. Distal tibial physis closure generally occurs by 16 years of age for boys and 14 for girls. Before this time the surrounding ligament structures can be stronger than the growth plates, which leads to a tendency for growth plate fractures over ankle sprains in this age group. Over a period of 18 months, the distal tibial physis closes first in the middle of the physis, then the medial part, and lastly the lateral side. A Tillaux fracture is a Salter-Harris III fracture of the anterolateral portion of the distal tibia, which results from an epiphyseal avulsion at the attachment site of the anterior talofibular ligament. Triplane fractures have fracture lines in 3 planes which involve the metaphyseal fragment posteriorly and an epiphyseal fragment which is more often lateral than medial. The patients fracture appears to involve the epiphysis and physis which is characteristic of a Salter-Harris Type III fracture. 1,2

     

    Answer D

    References

    1. Kay, Robert M. MD; Matthys, Gary A. MD. Pediatric Ankle Fractures: Evaluation and Treatment. Journal of the American Academy of Orthopaedic Surgeons 9(4):p 268-278, July 2001.
    2. Blackburn EW, Aronsson D, Rubright J, Lisle J. Current Concepts Review: Ankle fractures in Children. J Bone Joint Surg Am. 2012;94:1234-44
  29. Question 29 of 48
    29. Question

    An 8-year-old female is brought to the office by her mom with complaints of left elbow pain for 6 days. She injured the elbow from a fall off monkey bars and is having difficulty moving the elbow since the injury. On physical exam she has mild swelling around the elbow and tenderness to palpation over the olecranon and radial head. AP and lateral x-rays of the elbow taken in the ED the day of injury show fractures of the olecranon and radial neck. What is the best treatment option?

    Correct

    Pediatric elbow injuries are one of the most common reasons for emergency room or urgent care visits. There are 6 growth centers around the elbow which can make interpreting x-rays difficult in pediatric age groups. Comparison x-rays of the un-injured elbow are often necessary to distinguish normal and abnormal findings. Supracondylar fractures are the most common fracture around the elbow (60%), followed by lateral and medial condylar fractures, and less common, radial head and olecranon fractures. Non-displaced fractures are treated non-operatively with a 3-4 week period of immobilization. Immobilizing options generally include a long arm cast, removable long arm splint, or a sling. The use of a long arm cast or posterior splint provides improved pain relief over a sling in the first two weeks from injury. Elbow immobilization should not be continued beyond 4 weeks as arthrofibrosis of the elbow can start to set in and could lead to a permanent loss of motion. Silva et al found that the use of a removable long arm soft cast (removable under parent supervision) for non-displaced elbow fractures provides a convenient option for patients while safely maintaining fracture alignment. 1,2
    Answer C.
    References
    1. Lins RE, Simovitch RW, Waters PM. Pediatric elbow trauma. Orthopedic Clinics of North America. 1999 Jan 1;30(1):119-32.
    2. Silva M, Sadlik G, Avoian T, Ebramzadeh E. A removable long-arm soft cast to treat nondisplaced pediatric elbow fractures: a randomized, controlled trial. Journal of Pediatric Orthopaedics. 2018 Apr 1;38(4):223-9.

    Incorrect

    Pediatric elbow injuries are one of the most common reasons for emergency room or urgent care visits. There are 6 growth centers around the elbow which can make interpreting x-rays difficult in pediatric age groups. Comparison x-rays of the un-injured elbow are often necessary to distinguish normal and abnormal findings. Supracondylar fractures are the most common fracture around the elbow (60%), followed by lateral and medial condylar fractures, and less common, radial head and olecranon fractures. Non-displaced fractures are treated non-operatively with a 3-4 week period of immobilization. Immobilizing options generally include a long arm cast, removable long arm splint, or a sling. The use of a long arm cast or posterior splint provides improved pain relief over a sling in the first two weeks from injury. Elbow immobilization should not be continued beyond 4 weeks as arthrofibrosis of the elbow can start to set in and could lead to a permanent loss of motion. Silva et al found that the use of a removable long arm soft cast (removable under parent supervision) for non-displaced elbow fractures provides a convenient option for patients while safely maintaining fracture alignment. 1,2
    Answer C.
    References
    1. Lins RE, Simovitch RW, Waters PM. Pediatric elbow trauma. Orthopedic Clinics of North America. 1999 Jan 1;30(1):119-32.
    2. Silva M, Sadlik G, Avoian T, Ebramzadeh E. A removable long-arm soft cast to treat nondisplaced pediatric elbow fractures: a randomized, controlled trial. Journal of Pediatric Orthopaedics. 2018 Apr 1;38(4):223-9.

  30. Question 30 of 48
    30. Question

    A fracture through the epiphysis and physis, but sparring the metaphysis is what Salter-Harris type fracture?

    Correct

    Physeal injuries have many different classifications but the most commonly used and documented is the Salter-Haris Classification. In 1963 Dr. Robert Salter and Dr. J.J. Harris described this classification system. The system describes fractures involving the physis and is broken into 5 types based on involvement of the metaphysis, physis, and epiphysis.

    Type I – Involves separation of the epiphysis from the physis with the reproductive cells of the physis remaining attached to the epiphysis. These account for approximately 6% of all physeal fractures. Often there are no radiographic changes seen, thus requiring a thorough clinical exam to confirm this diagnosis.  The hallmark on the physical exam is point tenderness over the physis.  Any child presenting with these symptoms and a normal radiograph should pose a high suspicion for type 1 injury. Treatment consists of immobilization, ice, elevation and re examination in 1-2 weeks. Fortunately these injuries rarely result in growth disturbances and prognosis is excellent with proper treatment.

    •Type II – This is the most common type of physeal fracture accounting for approximately 75% of all physeal injuries. The fracture line extends along the weakest zone of the growth plate to the hypertrophic cell zone, then out through a portion of the metaphysis. There is a triangular shaped section of the metaphysis that often accompanies this fracture, and is referred to as the “Thurston-Holland” fragment. The germinal layer that is attached to the epiphysis is not involved therefore growth disturbances are rare. Treatment should consists of gentle reduction as not to cause further damage to the growth plate and immobilization.  Excessive attempts at a closed reduction may lead to growth plate damage and increase the risk of early physeal closure.

    Type III – These extend from the joint surface through the epiphysis and across the physis. They account for approximately 10% of physeal fractures. The prognosis with these is good but more guarded and is related to the preservation of the blood supply of the epiphyseal bone. Any fracture with significant displacement or fragmentation will have a significantly increased risk of damage to the blood supply.  These fractures typically occur in older children where the potential for remodeling is much less.  Therefore treatment requires an anatomical reduction of the joint surface, as to reduce the risk of long term sequelae.

    Type IV – The fracture line extends from the joint surface through the epiphysis, across the full thickness of the growth plate and through a portion of the metaphysis.  These account for approximately 10% of all physeal injuries.These fractures have an intra-articular component, and carry a high risk of growth arrest similar to type III fractures.  Treatment is similar to type III fractures, requiring an anatomic reduction of the joint surface.  These fractures are more likely to require operative intervention to achieve anatomical reduction, and the risk of growth disturbance is dependent on the blood supply disruption through the epiphysis.

    Type V – These are crush injuries of the physeal plate that are usually produced by compressive forces. The resultant axial compression crushes the physis, and specifically injures the cells of the reserve and proliferative zones. These are by far the most likely to cause growth arrest. There is typically minimal or no displacement of the epiphysis thereby making radiographic diagnosis difficult, although you may see narrowing of the physis radiographically. Injury is commonly diagnosed retrospectively once a bone growth abnormality has been identified.

    References
    Menckhoff C, Welter C, Sessums C. Pediatric wrist and elbow injuries: Mecha-nisms of injury, findings, and treatment. Pediatr Emerg Med Reports. 2008; 8:93-108

    Perron A, Miller M, Brady W.  Orthopedic Pitfalls in the ED: Pediatric Growth Plate Injuries. Am J Emerg Med.  2002; 20:50-54

     

    Incorrect

    Physeal injuries have many different classifications but the most commonly used and documented is the Salter-Haris Classification. In 1963 Dr. Robert Salter and Dr. J.J. Harris described this classification system. The system describes fractures involving the physis and is broken into 5 types based on involvement of the metaphysis, physis, and epiphysis.

    Type I – Involves separation of the epiphysis from the physis with the reproductive cells of the physis remaining attached to the epiphysis. These account for approximately 6% of all physeal fractures. Often there are no radiographic changes seen, thus requiring a thorough clinical exam to confirm this diagnosis.  The hallmark on the physical exam is point tenderness over the physis.  Any child presenting with these symptoms and a normal radiograph should pose a high suspicion for type 1 injury. Treatment consists of immobilization, ice, elevation and re examination in 1-2 weeks. Fortunately these injuries rarely result in growth disturbances and prognosis is excellent with proper treatment.

    •Type II – This is the most common type of physeal fracture accounting for approximately 75% of all physeal injuries. The fracture line extends along the weakest zone of the growth plate to the hypertrophic cell zone, then out through a portion of the metaphysis. There is a triangular shaped section of the metaphysis that often accompanies this fracture, and is referred to as the “Thurston-Holland” fragment. The germinal layer that is attached to the epiphysis is not involved therefore growth disturbances are rare. Treatment should consists of gentle reduction as not to cause further damage to the growth plate and immobilization.  Excessive attempts at a closed reduction may lead to growth plate damage and increase the risk of early physeal closure.

    Type III – These extend from the joint surface through the epiphysis and across the physis. They account for approximately 10% of physeal fractures. The prognosis with these is good but more guarded and is related to the preservation of the blood supply of the epiphyseal bone. Any fracture with significant displacement or fragmentation will have a significantly increased risk of damage to the blood supply.  These fractures typically occur in older children where the potential for remodeling is much less.  Therefore treatment requires an anatomical reduction of the joint surface, as to reduce the risk of long term sequelae.

    Type IV – The fracture line extends from the joint surface through the epiphysis, across the full thickness of the growth plate and through a portion of the metaphysis.  These account for approximately 10% of all physeal injuries.These fractures have an intra-articular component, and carry a high risk of growth arrest similar to type III fractures.  Treatment is similar to type III fractures, requiring an anatomic reduction of the joint surface.  These fractures are more likely to require operative intervention to achieve anatomical reduction, and the risk of growth disturbance is dependent on the blood supply disruption through the epiphysis.

    Type V – These are crush injuries of the physeal plate that are usually produced by compressive forces. The resultant axial compression crushes the physis, and specifically injures the cells of the reserve and proliferative zones. These are by far the most likely to cause growth arrest. There is typically minimal or no displacement of the epiphysis thereby making radiographic diagnosis difficult, although you may see narrowing of the physis radiographically. Injury is commonly diagnosed retrospectively once a bone growth abnormality has been identified.

    References
    Menckhoff C, Welter C, Sessums C. Pediatric wrist and elbow injuries: Mecha-nisms of injury, findings, and treatment. Pediatr Emerg Med Reports. 2008; 8:93-108

    Perron A, Miller M, Brady W.  Orthopedic Pitfalls in the ED: Pediatric Growth Plate Injuries. Am J Emerg Med.  2002; 20:50-54

  31. Question 31 of 48
    31. Question

    Which choice is not a component of the “female athlete triad”?

    Correct

    The female athlete triad includes a history of amenorrhea or oligomenorrhea, inadequate nutritional intake or eating disorders, and low bone density. Athletic participation that requires significant energy expenditure and low body weight puts women at an increased risk of the triad. When dietary intake is intentionally or unintentionally restricted energy expenditure may exceed calorie intake. As a result a cascade of physiological and neuroendocrine adaptations occur. Energy deficits or physiological stress may cause the disruption of gonadotropin-releasing hormone (GnRH) from the hypothalamus. GnRH pulses signal the pituitary gland to release luteinizing hormone (LH) and follicle stimulating hormone (FSH).  LH and FSH act on the ovaries to produce estrogen and progesterone for normal menstruation. Oligomenorrhea or amenorrhea and infertility can occur when LH and FSH production is disrupted.

    Menstruation and estrogen production significantly effects bone quality. Estrogen is an inhibitor of osteoclastic-mediated bone resorption. With a decline in estrogen levels osteclastic activity outpaces osteoblastic activity leading to increased calcium resporption and low bone density over time. As a result, amenorrheic or oligomenorrheic athletes have a higher incidence of stress fractures compared to athletes with normal menstrual cycles. The American College of Sports Medicine (ACSM) defines low BMD in premenopausal girls and women as a history of nutritional deficiencies, low estrogen, stress fractures, and/or other clinical risk factors for fracture together with a BMD Z-score between -1 and -2.0.  ACSM’s definition of osteoporosis includes all of the aforementioned risk factors with a BMD Z-score below -2.0.

    Although depression is a significant risk factor for eating disorders it is not one of the three components of the female athlete triad.

    References
    Female Athete Triad. http://nurse-practitioners-and-physician-assistants.advanceweb.com/continuing-education/ce-articles/female-athlete-triad.aspx?CP=2. 11/12/14.

    Female Athete Triad. http://www.femaleathletetriad.org/for-professionals/what-is-the-triad/. 11/12/14.

    Incorrect

    The female athlete triad includes a history of amenorrhea or oligomenorrhea, inadequate nutritional intake or eating disorders, and low bone density. Athletic participation that requires significant energy expenditure and low body weight puts women at an increased risk of the triad. When dietary intake is intentionally or unintentionally restricted energy expenditure may exceed calorie intake. As a result a cascade of physiological and neuroendocrine adaptations occur. Energy deficits or physiological stress may cause the disruption of gonadotropin-releasing hormone (GnRH) from the hypothalamus. GnRH pulses signal the pituitary gland to release luteinizing hormone (LH) and follicle stimulating hormone (FSH).  LH and FSH act on the ovaries to produce estrogen and progesterone for normal menstruation. Oligomenorrhea or amenorrhea and infertility can occur when LH and FSH production is disrupted.

    Menstruation and estrogen production significant effects bone quality. Estrogen is an inhibitor of osteoclastic-mediated bone resorption. With a decline in estrogen levels osteclastic activity outpaces osteoblastic activity leading to increase calcium resporption and low bone density over time. As a result, amenorrheic or oligomenorrheic athletes have a higher incidence of stress fractures compared to athletes with normal menstrual cycles. The American College of Sports Medicine (ACSM) defines low BMD in premenopausal girls and women as a history of nutritional deficiencies, low estrogen, stress fractures, and/or other clinical risk factors for fracture together with a BMD Z-score between -1 and -2.0.  ACSM’s definition of osteoporosis includes all of the aforementioned risk factors with a BMD Z-score below -2.0.

    Although depression is a significant risk factor for eating disorders it is not one of the three components of the female athlete triad.

    References
    Female Athete Triad. http://nurse-practitioners-and-physician-assistants.advanceweb.com/continuing-education/ce-articles/female-athlete-triad.aspx?CP=2. 11/12/14.

    Female Athete Triad. http://www.femaleathletetriad.org/for-professionals/what-is-the-triad/. 11/12/14.

  32. Question 32 of 48
    32. Question

    SCFE

    An 11-year old female presents to the office with a one month history of left hip pain. The pain started after dance class one month ago and has continued since.  The patient and mother thought the pain was a muscle strain and did not seek medical attention until now.  On exam the patient walks with a limp supporting the left leg. She is tender to palpation over the left thigh and is unable to internally rotate the hip actively. Passive external rotation causes pain as well.  AP pelvis and frog leg lateral x-rays are obtained (Figures 1 and 2 above). What is the best choice of treatment for this patient?

    Correct

    Slipped capital femoral epiphysis (SCFE) should always be considered when adolescent patients present with hip pain and difficulty bearing weight. Early detection and treatment is essential to avoid long-term complications such as avascular necrosis and early degenerative joint disease. Exam and radiographic findings for SCFE may be subtle so a thorough evaluation is necessary.

    SCFE typically occurs in adolescent males aged 13 to 15 years old with a 2:1 male to female ratio.  This age group is associated with the period of maximal skeletal growth (peak height velocity).  Rapid growth during puberty results in a period of decreased physeal strength.  Physiological loading on a weakened physis results in an increased risk of developing SCFE. Obese patients are at an increased risk secondary to additional physiological loading on the physis. The hips may be affected unilaterally or bilaterally. SCFE occurs bilaterally over 25% of the time with both slips generally occurring within the same 6 month period. The most common presentations for SCFE include: hip or groin pain, thigh pain, knee pain, limp, and decreased range of motion of the affected hip, often with no history of trauma.  In patients who present with SCFE, 60% of children are in the 95th percentile for body mass. A high index of suspicion must be maintained in patients with Down Syndrome, a history of endocrine abnormalities, or a slip under the age of 10 years. Endocrinopathies account for 5-8% of all slips with the most common being hypothyroidsim, panhypopituitarism, growth hormone abnormalities, and hypogonadism. An endocrine work-up should be done when SCFE occurs in patients younger than 10 years of age and older than 15 years of age. Patients with renal osteodystrophy have a higher rate of SCFE due to secondary hyperparathyroidism.

    Classification
    Clinical classification of SCFE is dependent on the duration of symptoms when the patient presents to clinic.  Acute slips are characterized by duration of symptoms  for less than 3 weeks. Chronic slips are classified by symptoms for greater than 3 weeks. Acute-on-chronic slips have had symptoms for at least 3 weeks with a new shorter duration of increased pain. Most slips show slow progressive displacement, but if left untreated, can cause significant disability. Another classification system is based on physeal stability and the predictability of developing osteonecrosis. Patients are determined to have unstable SCFE if they cannot bear weight on the affected extremity, with or without crutches, and stable SCFE if they can bear weight. Osteonecrosis was found to develop in nearly 50% of patients with unstable SCFE and in no patients classified as having stable SCFE.

    Describing displacement in a SCFE requires understanding that the head does not move, the Salter I fracture of the physis allows the femoral neck to rotate from under the capital epiphysis.  Understanding the extra-osseous and intra-osseous blood supply to the proximal femur is critical to help minimize the frequency of complications. The extra-osseous blood supply consists of the medial and lateral femoral circumflex arteries which penetrate the hip capsule and supply the femoral head, neck and trochanter.  Epiphyseal branches of these vessels cross the physis into the epiphysis.  When a SCFE occurs these vessels may be violated which puts the affected hip at risk for osteonecrosis.

    Plain anteroposterior and frog leg lateral radiographs of both hips are the gold standard to confirming the diagnosis of SCFE.  SCFE is best seen on frog leg lateral radiograph. In most cases SCFE can be diagnosed on plain films. A parallel line draw along the lateral femoral neck (Klein’s Line) should intersect a small amount of the epiphysis. When Klein’s line doesn’t intersect some of the physis, SCFE should be suspected. Radiographic appearance of SCFE may show the femoral epiphysis slipped slightly medially,  inferiorly, and posteriorly. Patients with chronic SCFE may have new bone formation on the inferior medial femoral neck.  A slip may not be seen on initial x-rays and an MRI can help diagnose SCFE in the pre-slip period. MRI findings may include physeal widening and edema. Techntium-99 Bone scan and MRI can be used to detect avascular necrosis (AVN) with MRI remaining highly specific for AVN. MRI can also be helpful to rule out other conditions in the differential including tumors, infections, subtle fractures, or surrounding soft tissue injuries. Ultrasound is seldom used for diagnosing SCFE. Initial laboratory work-up may include labs to rule out septic joint, chemistry panel to rule out renal failure, thyroid panel for endocrine abnormalities, and lyme titers to rule out lyme disease.

    Treatment
    In situ cannulated hip screw fixation is the preferred treatment for SCFE. Previous treatment with hip spica casting has largely been discontinued due its associated high complication rates and to the success of in situ screw fixation. The goal of treatments is to prevent OA by maintaining femoral head/acetabulum positioning, maintain blood supply of the femoral head preventing osteonecrosis, and eliminating pain acutely.   Patients with stable slips may begin  to use a walker or crutches with partial weight bearing restrictions for 6 weeks postoperatively. At 6 weeks the pain should resolve completely.  Athletic activities are resumed after 3 months.  Physeal closure occurs 6-12 months after surgery. Screw removal is not routinely performed or recommended.

    A postoperative CT scan is useful in the assessment of persistant pain. CT can show whether screw or pin fixation has penetrated the hip joint after fixation.   It also confirms closure of the physis if recurrent pain is an issue in a previously treated hip.  The risk of developing AVN and early chondrolysis is signifcantly greater in patients who presented with unstable SCFE preoperatively. For severe forms of SCFE, treatment options may include primary femoral osteotomy, bone graft epiphysiodesis, femoral neck osteoplasty, valgus derotational osteotomy and propylactic fixation of the contralateral hip.

    Conclusion:
    Children between the ages of 10 and 16-years old  who present to your clinic with a limp and pain in the groin, hip, thigh, or knee should be considered to have a SCFE until proven otherwise.  Remember, pulled groin muscles are rarely correct in children, although this common misdiagnosis is still made.  Clinicians should be aware of the endocrine abnormalities associated with SCFE and the need to manage comorbidities or seek consultation when appropriate.

    REFERENCES
    Weber E, Naujoks R, Smith BG. Slipped Capital Femoral Epiphysis: Orthopedic Knowledge Online Journal 2008.

    Kay, R.M. (2006). Slipped capital femoral epiphysis. In R.T. Morrissy & S.L. Weinstein (Eds.),  Lovell and Winter’s pediatric orthopaedics (pp. 1085-1116).  Philadelphia, PA:  Lippincott   Williams & Wilkins

    Aronsson DD, Loder RT, Breur GJ, Weinstein SL. Slipped Capital Femoral Ephiphysis: Current concepts. JAAOS. 2006; 14: 666-679

    Incorrect

    Slipped capital femoral epiphysis (SCFE) should always be considered when adolescent patients present with hip pain and difficulty bearing weight. Early detection and treatment is essential to avoid long-term complications such as avascular necrosis and early degenerative joint disease. Exam and radiographic findings for SCFE may be subtle so a thorough evaluation is necessary.

    SCFE typically occurs in adolescent males aged 13 to 15 years old with a 2:1 male to female ratio.  This age group is associated with the period of maximal skeletal growth (peak height velocity).  Rapid growth during puberty results in a period of decreased physeal strength.  Physiological loading on a weaken physis results in an increased risk of developing SCFE. Obese patients are at an increased risk secondary to addition physiological loading on the physis. The hips may be affected unilaterally or bilaterally. SCFE occurs bilaterally over 25% of the time with both slips generally occurring within the same 6 month period. The most common presentations for SCFE include: hip or groin pain, thigh pain, knee pain, limp, and decreased range of motion of the affected hip, often with no history of trauma.  In patients who present with SCFE, 60% of children are in the 95th percentile for body mass. A high index of suspicion must be maintained in patients with Down Syndrome, a history of endocrine abnormalities, or a slip under the age of 10 years. Endocrinopathies account for 5-8% of all slips with the most common being hypothyroidsim, panhypopituitarism, growth hormone abnormalities, and hypogonadism. An endocrine work-up should be done when SCFE occurs in patients younger than 10 years of age and older than 15 years of age. Patients with renal osteodystrophy have a higher rate of SCFE due to secondary hyperparathyroidism.

    Classification
    Clinical classification of SCFE is dependent on the duration of symptoms when the patient presents to clinic.  Acute slips are characterized by duration of symptoms  for less than 3 weeks. Chronic slips are classifed by symptoms for greater than 3 weeks. Acute-on-chronic slips have had symptoms for at least 3 weeks with a new shorter duration of increased pain. Most slips show slow progressive displacement, but if left untreated, can cause significant disability. Another classification system is based on physeal stability and the predictability of developing osteonecrosis. Patients are determined to have unstable SCFE if they cannot bear weight on the affected extremity, with or without crutches, and stable SCFE if they can bear weight. Osteonecrosis was found to develop in nearly 50% of patients with unstable SCFE and in no patients classified as having stable SCFE.

    Describing displacement in a SCFE requires understanding that the head does not move, the Salter I fracture of the physis allows the femoral neck to rotate from under the capital epiphysis.  Understanding the extra-osseous and intra-osseous blood supply to the proximal femur is critical to help minimize the frequency of complications. The extra-osseous blood supply consists of the medial and lateral femoral circumflex arteries which penetrate the hip capsule and supply the femoral head, neck and trochanter.  Epiphyseal branches of these vessels cross the physis into the epiphysis.  When a SCFE occurs these vessels may be violated which puts the affected hip at risk for osteonecrosis.

    Plain anteroposterior and frog leg lateral radiographs of both hips are the gold standard to confirming the diagnosis of SCFE.  SCFE is best seen on frog leg lateral radiograph. In most cases SCFE can be diagnosed on plain films. A paralell line draw along the lateral femoral neck (Klein’s Line) should intersect a small amount of the epiphysis. When Klein’s line doesn’t intersect some of the physis, SCFE should be suspected. Radiographic appearance of SCFE may show the femoral epiphysis slipped slightly medially,  inferiorly, and posteriorly. Patients with chronic SCFE may have new bone formation on the inferior medial femoral neck.  A slip may not be seen on initial x-rays and an MRI can help diagnose SCFE in the pre-slip period. MRI findings may include physeal widening and edema. Techntium-99 Bone scan and MRI can be used to detect avascular necrosis (AVN) with MRI remaining highly specific for AVN. MRI can also be helpful to rule out other conditions in the differential including tumors, infections, subtle fractures, or surrounding soft tissue injuries. Ultrasound is seldom used for diagnosing SCFE. Initial laboratory work-up may include labs to rule out septic joint, chemistry panel to rule out renal failure, thyroid panel for endocrine abnormalities, and lyme titers to rule out lyme disease.

    Treatment
    In situ cannulated hip screw fixation is the preferred treatment for SCFE. Previous treatment with hip spica casting has largely been discontinued due its associated high complication rates and to the success of in situ screw fixation. The goal of treatments is to prevent OA by maintaining femoral head/acetabulum positioning, maintain blood supply of the femoral head preventing osteonecrosis, and eliminating pain acutely.   Patients with stable slips may begin  to use a walker or crutches with partial weight bearing restrictions for 6 weeks postoperatively. At 6 weeks the pain should resolve completely.  Athletic activities are resumed after 3 months.  Physeal closure occurs 6-12 months after surgery. Screw removal is not routinely performed or recommended.

    A postoperative CT scan is useful in the assessment of persistant pain. CT can show whether screw or pin fixation has penetrated the hip joint after fixation.   It also confirms closure of the physis if recurrent pain is an issue in a previously treated hip.  The risk of developing AVN and early chondrolysis is signifcantly greater in patients who presented with unstable SCFE preoperatively. For severe forms of SCFE, treatment options may include primary femoral osteotomy, bone graft epiphysiodesis, femoral neck osteoplasty, valgus derotational osteotomy and propylactic fixation of the contralateral hip.

    Conclusion:
    Children between the ages of 10 and 16-years old  who present to your clinic with a limp and pain in the groin, hip, thigh, or knee should be considered to have a SCFE until proven otherwise.  Remember, pulled groin muscles are rarely correct in children, although this common misdiagnosis is still made.  Clinicians should be aware of the endocrine abnormalities associated with SCFE and the need to manage comorbidities or seek consultation when appropriate.

    REFERENCES
    Weber E, Naujoks R, Smith BG. Slipped Capital Femoral Epiphysis: Orthopedic Knowledge Online Journal 2008.

    Kay, R.M. (2006). Slipped capital femoral epiphysis. In R.T. Morrissy & S.L. Weinstein (Eds.),  Lovell and Winter’s pediatric orthopaedics (pp. 1085-1116).  Philadelphia, PA:  Lippincott   Williams & Wilkins

    Aronsson DD, Loder RT, Breur GJ, Weinstein SL. Slipped Capital Femoral Ephiphysis: Current concepts. JAAOS. 2006; 14: 666-679

  33. Question 33 of 48
    33. Question

    An 8 year-old boy presents to the emergency department with right hip pain for 2 days. The boy presents with his mom who denies any recent injury. The boy is unable to put any weight on the right leg and has severe pain with any passive motion of the hip. The boy is afebrile and his ESR and CRP are both within normal limits. He has a slightly elevated WBC. His mom admits he is very healthy other than an upper respiratory illness one week ago. AP radiograph of the right hip is normal. What is the most likely diagnosis?

    Correct

    Pediatric hip pain is a challenging presentation that can be seen in many healthcare settings. The differential diagnosis in an 8 year old who presents with acute hip pain and an elevated WBC includes either septic arthritis vs. transient synovitis. These two conditions are often very difficult to differentiate. Transient synovitis of the hip often presents with little to no WBC elevation and normal ESR and CRP labs. The etiology of transient synovitis is unknown but usually presents with a history of a recent upper respiratory infection. Pediatric hip pain patients that present with a fever and elevated labs including a WBC, ESR, and CRP more commonly have septic arthritis. The CRP is the most predictable lab to diagnose septic arthritis. With an acute infection, the CRP will rise within 6 hours of an inflammatory response and peaks at two days. A CRP > 20 is highly predictive of septic arthritis. Radiographs of the hip should always be obtained to rule out bony injury or deformity. Legg-Calve-Perthes disease is an idiopathic avascular necrosis of the femoral epiphyses that occurs in children most commonly between the ages of 4-8. Legg-Calve-Perthes will often show irregularities of the femoral head on radiographs. Slipped Capital Femoral Epiphysis (SCFE) is a slippage of the metaphysis relative to the epiphysis and most commonly occurs in obese males around the age of 13. 1,2
    Answer B.
    References
    1. Arkader A, Brusalis C, Warner WC, Conway JH, Noonan K. Update in Pediatric Musculoskeletal Infections: When It Is, When It Isn’t, and What to Do. JAAOS 2016 – Volume 24 – Issue 9 – p e112-e121
    2. Mininder K, Zurakowski D, Kasser J. Differentiating Between Septic Arthritis and Transient Synovitis of the Hip in Children: An Evidence-Based Clinical Prediction Algorithm. JBJS 1999 – Volume 81 – Issue 12 – p. 1662-70

    Incorrect

    Pediatric hip pain is a challenging presentation that can be seen in many healthcare settings. The differential diagnosis in an 8 year old who presents with acute hip pain and an elevated WBC includes either septic arthritis vs. transient synovitis. These two conditions are often very difficult to differentiate. Transient synovitis of the hip often presents with little to no WBC elevation and normal ESR and CRP labs. The etiology of transient synovitis is unknown but usually presents with a history of a recent upper respiratory infection. Pediatric hip pain patients that present with a fever and elevated labs including a WBC, ESR, and CRP more commonly have septic arthritis. The CRP is the most predictable lab to diagnose septic arthritis. With an acute infection, the CRP will rise within 6 hours of an inflammatory response and peaks at two days. A CRP > 20 is highly predictive of septic arthritis. Radiographs of the hip should always be obtained to rule out bony injury or deformity. Legg-Calve-Perthes disease is an idiopathic avascular necrosis of the femoral epiphyses that occurs in children most commonly between the ages of 4-8. Legg-Calve-Perthes will often show irregularities of the femoral head on radiographs. Slipped Capital Femoral Epiphysis (SCFE) is a slippage of the metaphysis relative to the epiphysis and most commonly occurs in obese males around the age of 13. 1,2
    Answer B.
    References
    1. Arkader A, Brusalis C, Warner WC, Conway JH, Noonan K. Update in Pediatric Musculoskeletal Infections: When It Is, When It Isn’t, and What to Do. JAAOS 2016 – Volume 24 – Issue 9 – p e112-e121
    2. Mininder K, Zurakowski D, Kasser J. Differentiating Between Septic Arthritis and Transient Synovitis of the Hip in Children: An Evidence-Based Clinical Prediction Algorithm. JBJS 1999 – Volume 81 – Issue 12 – p. 1662-70

  34. Question 34 of 48
    34. Question

    A 9-year-old presents to the emergency room with left arm pain after falling off a trampoline earlier in the day. She noticed immediate pain, weakness, and deformity to the left arm after the fall. AP x-ray taken in the ED shows a displaced proximal one-third humerus fracture. On physical exam the patients skin and neurovascular status distal to the fracture are both intact. What is the best initial treatment option for this patient?

    Correct

    Pediatric humeral shaft fractures are almost always treated nonoperatively.  Pediatric patients have excellent remodeling potential which will straighten the fracture over 6-8 weeks of immobilization.  Conservative treatments include a coaptation splint, functional humeral brace like the Sarmiento or Galveston brace, simple sling, and a hanging cast for a period of 6 weeks. This patient was treated in a sling that allowed gravity to help reduce the fracture. The sling should be worn loosely to allow the arm to “dangle” in the sling. A sling that pulls the arm up proximally will angulate the fracture. Coaptation splints are often used in the emergency department for humeral shaft fractures as they provide excellent immobilization of the entire arm. A Sarmiento brace is a rigid shell with adhesive straps that wrap circumferentially around the arm at the fracture site. This provides soft tissue compression at the fracture site which helps reduce the fracture. A functional humeral brace like the Sarmiento brace allows for range of motion of the elbow when the patient tolerates. A hanging cast may be used for shortened oblique fractures to provide extra gravitational weight to reduce the fracture. The hanging cast should extend 2 cm proximal to the fracture so it is generally used for more distal fractures. Typically, a coaptation splint is placed in the emergency room with a sling for initial immobilization of humeral shaft fractures. At 1-2 weeks post fracture a function humeral brace or sling alone can be used. 1,2

    Answer B.

    References

    1. O’Shaughnessy MA, Parry JA, Liu H, Stans AA, Larson AN, Milbrandt TA. Management of paediatric humeral shaft fractures and associated nerve palsy. J Child Orthop. 2019;13(5):508-515. doi:10.1302/1863-2548.13.190012
    2. Pediatric humeral shaft fractures. http://www.orthobullets.com. Accessed on 11/21/20.
    Incorrect

    Pediatric humeral shaft fractures are almost always treated nonoperatively.  Pediatric patients have excellent remodeling potential which will straighten the fracture over 6-8 weeks of immobilization.  Conservative treatments include a coaptation splint, functional humeral brace like the Sarmiento or Galveston brace, simple sling, and a hanging cast for a period of 6 weeks. This patient was treated in a sling that allowed gravity to help reduce the fracture. The sling should be worn loosely to allow the arm to “dangle” in the sling. A sling that pulls the arm up proximally will angulate the fracture. Coaptation splints are often used in the emergency department for humeral shaft fractures as they provide excellent immobilization of the entire arm. A Sarmiento brace is a rigid shell with adhesive straps that wrap circumferentially around the arm at the fracture site. This provides soft tissue compression at the fracture site which helps reduce the fracture. A functional humeral brace like the Sarmiento brace allows for range of motion of the elbow when the patient tolerates. A hanging cast may be used for shortened oblique fractures to provide extra gravitational weight to reduce the fracture. The hanging cast should extend 2 cm proximal to the fracture so it is generally used for more distal fractures. Typically, a coaptation splint is placed in the emergency room with a sling for initial immobilization of humeral shaft fractures. At 1-2 weeks post fracture a function humeral brace or sling alone can be used. 1,2

    Answer B.

    References

    1. O’Shaughnessy MA, Parry JA, Liu H, Stans AA, Larson AN, Milbrandt TA. Management of paediatric humeral shaft fractures and associated nerve palsy. J Child Orthop. 2019;13(5):508-515. doi:10.1302/1863-2548.13.190012
    2. Pediatric humeral shaft fractures. http://www.orthobullets.com. Accessed on 11/21/20.
  35. Question 35 of 48
    35. Question

    salter fx

    A 13-year old male presents to your office one week after sustaining a right knee injury while jumping off a wall and landing awkwardly. He was seen in the emergency department after the injury and initial x-rays showed a possible distal femur physeal fracture (figure 1). CT was then performed showing a minimally displaced Salter-Harris type II distal femur fracture at the medial metaphysis with slight widening of the posterior lateral growth plate (figures 2 and 3).Which statement below is true regarding Salter-Harris type II distal femur fractures?

     

    Correct

    Distal femoral physeal fractures are particularly prone to growth disturbances, including partial or complete premature growth arrest. The distal femoral physis contributes to 40% of lower extremity growth so growth arrest can cause significant deformity and leg length discrepancy. This patient presents with a minimally displaced Salter-Harris type II fracture characterized by an oblique metaphyseal fracture line that stops at the physis. The metaphyseal corner that remains attached to the physis is called a Thurston-Holland Fragment, which can vary in size. The physis under the Thurston-Holland fragment is usually spared from injury and the contralateral side of the physis is more likely to experience growth arrest. This patient’s metaphyseal fracture occurred medially and therefore the patient is at risk of a valgus deformity should growth arrest occur. 1

    Non-displaced distal femoral physeal fractures and fractures with less than 2 mm of displacement can be treated with a well-molded long leg cast. The risk of growth arrest for Salter-Harris type II fractures is significant and close follow-up with serial x-rays is necessary. Leg length discrepancy or angular deformity as a result of physeal disturbances can occur in 30-60% of patients with Salter-Harris type II fractures. The implications of physeal arrest is largely dependent on age and remaining growth left at the time of injury. Patients with significant growth potential remaining are more likely to have a significant deformity. Radiographs should be followed for 12 to 24 months to ensure no growth arrest has occurred. 1,2

    References

    1. Distal femoral physeal fractures. http://www.orthobullets.com. Accessed on 12/11/2016.

    2. Skaggs DL. Extra-Articular Injuries of the Knee. Rockwood and Wilkins’ Fractures in Children 6th Edition. Lippincott Williams & Wilkins. Philadelphia, PA. 2006.

    Incorrect

    Distal femoral physeal fractures are particularly prone to growth disturbances, including partial or complete premature growth arrest. The distal femoral physis contributes to 40% of lower extremity growth so growth arrest can cause significant deformity and leg length discrepancy. This patient presents with a minimally displaced Salter-Harris type II fracture characterized by an oblique metaphyseal fracture line that stops at the physis. The metaphyseal corner that remains attached to the physis is called a Thurston-Holland Fragment, which can vary in size. The physis under the Thurston-Holland fragment is usually spared from injury and the contralateral side of the physis is more likely to experience growth arrest. This patient’s metaphyseal fracture occurred medially and therefore the patient is at risk of a valgus deformity should growth arrest occur. 1

    Non-displaced distal femoral physeal fractures and fractures with less than 2 mm of displacement can be treated with a well-molded long leg cast. The risk of growth arrest for Salter-Harris type II fractures is significant and close follow-up with serial x-rays is necessary. Leg length discrepancy or angular deformity as a result of physeal disturbances can occur in 30-60% of patients with Salter-Harris type II fractures. The implications of physeal arrest is largely dependent on age and remaining growth left at the time of injury. Patients with significant growth potential remaining are more likely to have a significant deformity. Radiographs should be followed for 12 to 24 months to ensure no growth arrest has occurred. 1,2

    References

    1. Distal femoral physeal fractures. http://www.orthobullets.com. Accessed on 12/11/2016.

    2. Skaggs DL. Extra-Articular Injuries of the Knee. Rockwood and Wilkins’ Fractures in Children 6th Edition. Lippincott Williams & Wilkins. Philadelphia, PA. 2006.

  36. Question 36 of 48
    36. Question

    bicorticol wrist

    An 8 year-old male presents to your clinic with right wrist pain after an injury 5 days ago. He was playing soccer with his older brothers when he caught a ball off the hand. He had immediate pain and was unable to continue playing. His parents brought him to an urgent care the next day where AP and lateral x-rays showed a distal radius fracture. What is the best initial treatment option for this patient?

     

    Correct

    Forearm fractures are the most common pediatric fracture and most of these fractures occur at the distal radius. The porous metaphyseal bone in the pediatric population is prone to compression injuries with minimal cortical disruption. Stable unicortical metaphyseal fractures proximal to the physis are referred to as buckle or torus fractures. Pediatric distal radius fractures are stable when there is unicortical involvement and can be unstable or prone to displacement when there is bicortical involvement. A bicortical fracture line seen on AP and lateral radiographs represents a more severe fracture than a stable unicortical buckle fracture. It is therefore critical to recognize whether the tension side of a metaphyseal fracture is intact to help guide treatment. This patient presents with a distal radius fracture with bicortical involvement. Initial treatment with a removable splint would result in an unacceptably high risk of fracture displacement. A short arm cast can be wiggled off by non-compliant kids and fails to restrict forearm rotation. An initial long arm cast offers the best protection for fractures that are prone to displacement. A long arm cast for 2-3 weeks, or until cortical bridging is seen on x-ray, is commonly used for bicortical distal radius fractures. Patients can then be transitioned to a short arm cast for an addition 2-3 weeks once cortical bridging is seen. Most distal radius fractures usually take 4-6 weeks to heal, depending on the patient’s age. 1, 2

    1. Waters PM, Mih AD. Fractures of the Distal Radius and Ulna. Rockwood and Wilkins’ Fractures in Children 6th Edition. Lippincott Williams & Wilkins. Philadelphia, PA. 2006.

    2. Torus Fracture. http://www.wheelessonlone.com. Accessed on 12/12/16.

    Incorrect

    Forearm fractures are the most common pediatric fracture and most of these fractures occur at the distal radius. The porous metaphyseal bone in the pediatric population is prone to compression injuries with minimal cortical disruption. Stable unicortical metaphyseal fractures proximal to the physis are referred to as buckle or torus fractures. Pediatric distal radius fractures are stable when there is unicortical involvement and can be unstable or prone to displacement when there is bicortical involvement. A bicortical fracture line seen on AP and lateral radiographs represents a more severe fracture than a stable unicortical buckle fracture. It is therefore critical to recognize whether the tension side of a metaphyseal fracture is intact to help guide treatment. This patient presents with a distal radius fracture with bicortical involvement. Initial treatment with a removable splint would result in an unacceptably high risk of fracture displacement. A short arm cast can be wiggled off by non-compliant kids and fails to restrict forearm rotation. An initial long arm cast offers the best protection for fractures that are prone to displacement. A long arm cast for 2-3 weeks, or until cortical bridging is seen on x-ray, is commonly used for bicortical distal radius fractures. Patients can then be transitioned to a short arm cast for an addition 2-3 weeks once cortical bridging is seen. Most distal radius fractures usually take 4-6 weeks to heal, depending on the patient’s age. 1, 2

    1. Waters PM, Mih AD. Fractures of the Distal Radius and Ulna. Rockwood and Wilkins’ Fractures in Children 6th Edition. Lippincott Williams & Wilkins. Philadelphia, PA. 2006.

    2. Torus Fracture. http://www.wheelessonlone.com. Accessed on 12/12/16.

  37. Question 37 of 48
    37. Question

    abuse

    A 4-month old infant girl presents to the emergency department with his parents after a full day of inconsolable crying. The father reported that she tripped while holding his child earlier in the day. He apparently lost his balance as the child fell to the ground and began crying. However, after a skeletal survey was performed and a spiral fracture of the left femur was identified, non-accidental trauma was determined. Femur x-rays are shown in figures 1 and 2 above. The patient was placed in child protective services and orthopedics was consulted. What is the best treatment option for this fracture?

     

    Correct

    Child abuse should be suspected in children who present with a femur fracture before the age of 5 years, and in particular, those occurring before walking age. Child abuse is the most common cause of femur fracture in children under 1 years-old. Femur fractures occurring between the ages of 0 to 6 months are relatively stable injuries because of the thick periosteum present in this age group. Fractures in infants have remarkable remodeling potential and heal within 3-4 weeks without complications or long-term sequelae. A Pavlik harness is the recommended treatment for femur fractures ages 0 to 6 months. A Pavlik harness places the hip in moderate flexion and abduction which helps align the distal fragment with the proximal fragment. The harness can be applied without anesthesia or sedation and the straps can be adjusted to help reduce the fracture. A Pavlik harness allows for diaper changes without removing and can be changed by the patient’s care giver should it become soiled. Excessive hip flexion in the harness should be avoided to prevent femoral nerve injury which may present as decreased quadriceps function. Hip spica casting may be considered in fractures that are significantly shortened or angulated in this age group. Femur fractures in patients 7 months to 5 years old without significant shortening are usually treated with early hip spica casting, while flexible intramedullary nails are used for the same fracture pattern in children 6 to 11 years of age. 1,2

    References

    1. Pediatric femur fractures. http://www.orthobullets.com. Accessed on 10/17/16.

    2. Beaty JH, Kasser JR. Rockwood and Wilkins’ Fractures in Children 6th Edition. Lippincott Williams & Wilkins. Philadelphia, PA. 2006.

    Incorrect

    Child abuse should be suspected in children who present with a femur fracture before the age of 5 years, and in particular, those occurring before walking age. Child abuse is the most common cause of femur fracture in children under 1 years-old. Femur fractures occurring between the ages of 0 to 6 months are relatively stable injuries because of the thick periosteum present in this age group. Fractures in infants have remarkable remodeling potential and heal within 3-4 weeks without complications or long-term sequelae. A Pavlik harness is the recommended treatment for femur fractures ages 0 to 6 months. A Pavlik harness places the hip in moderate flexion and abduction which helps align the distal fragment with the proximal fragment. The harness can be applied without anesthesia or sedation and the straps can be adjusted to help reduce the fracture. A Pavlik harness allows for diaper changes without removing and can be changed by the patient’s care giver should it become soiled. Excessive hip flexion in the harness should be avoided to prevent femoral nerve injury which may present as decreased quadriceps function. Hip spica casting may be considered in fractures that are significantly shortened or angulated in this age group. Femur fractures in patients 7 months to 5 years old without significant shortening are usually treated with early hip spica casting, while flexible intramedullary nails are used for the same fracture pattern in children 6 to 11 years of age. 1,2

    References

    1. Pediatric femur fractures. http://www.orthobullets.com. Accessed on 10/17/16.

    2. Beaty JH, Kasser JR. Rockwood and Wilkins’ Fractures in Children 6th Edition. Lippincott Williams & Wilkins. Philadelphia, PA. 2006.

  38. Question 38 of 48
    38. Question

    imperfecta

    An 8-year old girl presents to your office with left ankle pain after “stepping wrong” 2 days ago. She hasn’t been able to put full weight on the ankle since. The patient appears very small for her age and has a dysmorphic face, but is of normal intelligence for her age. Her mom states she has had over 30 fractures and several operations before this recent injury. X-rays of the tibia and fibula show a non-displaced fracture of the fibula. A bowed tibia is also seen on the lateral x-ray. Which condition is causing this deformity?

     

    Correct

    The patient presents with classic physical exam and past medical history findings of type III Osteogenesis Imperfecta (OI).  Findings of OI include extremely fragile bones with history of multiple fractures, short stature, bowing of long bones, blue sclera, abnormal skull shape and tooth formation, and hearing loss beginning in the second decade of life.  OI is a heritable disorder caused by a decrease in type 1 collagen which provides tensile strength to bones, skin, tendons, dentin, and sclera.

    The Sillence classification system describes 7 types of OI based on genetic and clinical criteria. Type I (mild) is autosomal dominant with clinical features of blue sclerae, mild bone fragility and joint laxity, and a higher incidence of hearing loss. Type I accounts for 50% of all patients with OI. Type II is the most severe form with fractures occurring at birth and is often lethal in the perinatal period.  Type I and II patients have very dark blue or gray sclerae.  Type III is the most severe form in patients who survive the neonatal period. Patients may have fractures at birth, frequent fractures throughout childhood, short stature, and a triangular facial shape. Type IV patients are moderately affected with fewer fractures and less growth retardation than type III.  Types I and IV are autosomal dominant where types II and III are autosomal recessive.  Types V, VI, and VII are similar to type IV with the exception of a few characteristic findings. Type V is associated with large hypertrophic callus formation after fracture.  Type VI is rare and can be differentiated from type IV by characteristic mineralization on bone biopsy. Type VII is associated with infantile white sclerae and coxa vera.

    OI is a clinical diagnosis based on genetics, presence of fractures, and other clinical features. Bisphosphonates are commonly used in treatment to help increase bone density, decrease fractures, and improve quality of life in OI patients. Surgical fixation with telescoping rods and realignment osteotomy procedures are used for fracture fixation and deformity correction. Fractures will become less frequent when skeletal growth slows and will often cease after puberty. However, most patients with OI will have osteoporosis in adulthood and require treatments to improve bone density.

    References

    1. Osteogenesis imperfect. http://www.oif.org. Accessed on 3/20/16.
    2. Burnei G, Vlad C, Goeorgescue I, Gavriliu TS, Dan D. Osteogenesis Imperfecta: Diagnosis and Treatment. JAAOS 2008; 16: 356-366.
    Incorrect

    The patient presents with classic physical exam and past medical history findings of type III Osteogenesis Imperfecta (OI).  Findings of OI include extremely fragile bones with history of multiple fractures, short stature, bowing of long bones, blue sclera, abnormal skull shape and tooth formation, and hearing loss beginning in the second decade of life.  OI is a heritable disorder caused by a decrease in type 1 collagen which provides tensile strength to bones, skin, tendons, dentin, and sclera.

    The Sillence classification system describes 7 types of OI based on genetic and clinical criteria. Type I (mild) is autosomal dominant with clinical features of blue sclerae, mild bone fragility and joint laxity, and a higher incidence of hearing loss. Type I accounts for 50% of all patients with OI. Type II is the most severe form with fractures occurring at birth and is often lethal in the perinatal period.  Type I and II patients have very dark blue or gray sclerae.  Type III is the most severe form in patients who survive the neonatal period. Patients may have fractures at birth, frequent fractures throughout childhood, short stature, and a triangular facial shape. Type IV patients are moderately affected with fewer fractures and less growth retardation than type III.  Types I and IV are autosomal dominant where types II and III are autosomal recessive.  Types V, VI, and VII are similar to type IV with the exception of a few characteristic findings. Type V is associated with large hypertrophic callus formation after fracture.  Type VI is rare and can be differentiated from type IV by characteristic mineralization on bone biopsy. Type VII is associated with infantile white sclerae and coxa vera.

    OI is a clinical diagnosis based on genetics, presence of fractures, and other clinical features. Bisphosphonates are commonly used in treatment to help increase bone density, decrease fractures, and improve quality of life in OI patients. Surgical fixation with telescoping rods and realignment osteotomy procedures are used for fracture fixation and deformity correction. Fractures will become less frequent when skeletal growth slows and will often cease after puberty. However, most patients with OI will have osteoporosis in adulthood and require treatments to improve bone density.

    References

    1. Osteogenesis imperfect. http://www.oif.org. Accessed on 3/20/16.
    2. Burnei G, Vlad C, Goeorgescue I, Gavriliu TS, Dan D. Osteogenesis Imperfecta: Diagnosis and Treatment. JAAOS 2008; 16: 356-366.
  39. Question 39 of 48
    39. Question

    hip aiis

    A 13-year old male presents to the office with right hip pain after being tackled during a football game two days ago. He had difficulty ambulating after the injury and was unable to continue playing. He presents to the office on crutches with continued right hip pain. X-rays taken in the office are shown above. What is the most appropriate treatment option for this patient’s injury?

    Correct

    This patients x-rays show an anterior inferior iliac spine (AIIS) avulsion fracture.  An AIIS avulsion fracture represents an apophyseal injury at the insertion site of the rectus femoris. This injury generally occurs with a sudden forceful extension of the hip causing a pull of the rectus femoris.  Patients often complain of feeling a “pop” during the injury and increased pain with hip flexion. Exam findings may include localized pain and tenderness over the AIIS. Increased pain is elicited with resistive hip flexion and knee extension. An AP pelvis x-ray should be obtained to confirm an avulsion fracture vs. a normal adolescent variant.   Treatment consists of partial weight bearing for 2-3 weeks on crutches then weight bearing as tolerated thereafter. The patient is instructed to avoid active hip flexion and passive hip extension for 2-3 weeks.  Physical therapy is initiated at 3 weeks with a focus on a gradual increase in strength training. Return to sports without restrictions can begin at 8 weeks if the patient is back to full strength.  Open reduction and internal fixation is rarely indicated with AIIS avulsion fractures; surgical indications may include displacement > 2 cm, painful nonunion, and exostosis formation.

    References
    Beaty JH, Kasser JR. Rockwood and Wilkin’s Fractures in Children. 6th Edition. Lippincott Williams and Wilkins. Philadelphia, PA. 2006.

    AIIS Avulsion Fracture. http://www.orthobullets.com/sports/3131/anterior-inferior-iliac-spine-avulsion-aiis. 11/15/2014.

    Incorrect

    This patients x-rays show an anterior inferior iliac spine (AIIS) avulsion fracture.  An AIIS avulsion fracture represents an apophyseal injury at the insertion site of the rectus femoris. This injury generally occurs with a sudden forceful extension of the hip causing a pull of the rectus femoris.  Patients often complain of feeling a “pop” during the injury and increased pain with hip flexion. Exam findings may include localized pain and tenderness over the AIIS. Increased pain is elicited with resistive hip flexion and knee extension. An AP pelvis x-ray should be obtained to confirm an avulsion fracture vs. a normal adolescent variant.   Treatment consists of partial weight bearing for 2-3 weeks on crutches then weight bearing as tolerated thereafter. The patient is instructed to avoid active hip flexion and passive hip extension for 2-3 weeks.  Physical therapy is initiated at 3 weeks with a focus on a gradual increase in strength training. Return to sports without restrictions can begin at 8 weeks if the patient is back to full strength.  Open reduction and internal fixation is rarely indicated with AIIS avulsion fractures; surgical indications may include displacement > 2 cm, painful nonunion, and exostosis formation.

    References
    Beaty JH, Kasser JR. Rockwood and Wilkin’s Fractures in Children. 6th Edition. Lippincott Williams and Wilkins. Philadelphia, PA. 2006.

    AIIS Avulsion Fracture. http://www.orthobullets.com/sports/3131/anterior-inferior-iliac-spine-avulsion-aiis. 11/15/2014.

  40. Question 40 of 48
    40. Question

    A 14 year girl presents to the office with left posterior buttock pain after a soccer injury 2 days ago. She is having trouble walking now and has pain from the buttock that radiates to the hamstring. AP pelvis shows a left ischial tuberosity avulsion fracture. The avulsion appears to be displaced 1.5 cm. What is the best treatment option?

    Correct

    The ischial tuberosity is an attachment site for the proximal hamstring tendons. The three muscles that make up the hamstring include the semimembranosus, the semitendinosus, and the biceps femoris. The hamstrings act to extend the hip and flex the knee. The hamstrings can be torn proximally, midsubstance, or distal. The most common site of rupture is the proximal myotendinous junction. Avulsion injuries are less common and are generally seen in skeletally immature patients. Hamstring injuries cause a sharp pain in the posterior buttock which can be associated with significant ecchymosis. All patients with suspected hamstring injuries should have an AP pelvis x-ray to look for an avulsion fracture of the ischial tuberosity. In general most all hamstring tendon injuries are treated non-operatively with a period of protective weight-bearing for 4-6 weeks. The hamstrings can tighten quickly after tearing so physical therapy should be initiated after 4 weeks to focus on hamstring stretches. Athletes are allowed to return to sports when the hamstring is 90% as strong as the contralateral side. Operative treatment is indicated when there are three tendons torn off the ischial tuberosity or there is >2 cm of displacement of an osseous avulsion. These patients generally have significant chronic weakness and pain when treated non-operatively. 1,2
    Answer B.
    References
    1. Hofman KJ, Paggi Adam, Conors D, Miller S. Complete Avulsion of the Proximal Hamstring Insertion: Functional Outcomes After Nonsurgical Treatment. The Journal Of Bone And Joint Surgery – Scientific Articles: 18 June 2014 – Volume 96 – Issue 12 – p. 1022-1025
    2. Alzahrani MM, Aldebeyan S, Abduljabbar F, Martineau PA. Hamstring Injuries in Athletes: Diagnosis and Treatment. Jbjs Reviews – Review Article: 30 June 2015 – Volume 3 – Issue 6 – p. e5

    Incorrect

    The ischial tuberosity is an attachment site for the proximal hamstring tendons. The three muscles that make up the hamstring include the semimembranosus, the semitendinosus, and the biceps femoris. The hamstrings act to extend the hip and flex the knee. The hamstrings can be torn proximally, midsubstance, or distal. The most common site of rupture is the proximal myotendinous junction. Avulsion injuries are less common and are generally seen in skeletally immature patients. Hamstring injuries cause a sharp pain in the posterior buttock which can be associated with significant ecchymosis. All patients with suspected hamstring injuries should have an AP pelvis x-ray to look for an avulsion fracture of the ischial tuberosity. In general most all hamstring tendon injuries are treated non-operatively with a period of protective weight-bearing for 4-6 weeks. The hamstrings can tighten quickly after tearing so physical therapy should be initiated after 4 weeks to focus on hamstring stretches. Athletes are allowed to return to sports when the hamstring is 90% as strong as the contralateral side. Operative treatment is indicated when there are three tendons torn off the ischial tuberosity or there is >2 cm of displacement of an osseous avulsion. These patients generally have significant chronic weakness and pain when treated non-operatively. 1,2
    Answer B.
    References
    1. Hofman KJ, Paggi Adam, Conors D, Miller S. Complete Avulsion of the Proximal Hamstring Insertion: Functional Outcomes After Nonsurgical Treatment. The Journal Of Bone And Joint Surgery – Scientific Articles: 18 June 2014 – Volume 96 – Issue 12 – p. 1022-1025
    2. Alzahrani MM, Aldebeyan S, Abduljabbar F, Martineau PA. Hamstring Injuries in Athletes: Diagnosis and Treatment. Jbjs Reviews – Review Article: 30 June 2015 – Volume 3 – Issue 6 – p. e5

  41. Question 41 of 48
    41. Question


    A 16 year old girl presents to your office with her mother with complaints of worsening bilateral lower extremity pain over the last 2 months. She has been running 6-8 miles every day over the last few months for her cross country running team. Over the last 3 weeks she has “trained a little harder than usual” due to an upcoming state tournament. She admits to a normal diet and has had regular periods each month for the past year. Her medications include an oral contraceptive to help with acne but otherwise she has no past medical history. Her primary care physician ordered a bone density test which was normal. Coronal MRI image (figure 1) shows bilateral tibial stress fractures. Which choice is the most likely cause of her stress fractures?

    Correct

    Excessive endurance training in athletics can alter hormone balances and have negative consequences on the structural integrity of bone. Skeletal bone is constantly remodeled and repaired in response to stress placed during weight bearing activities. Bone is resorbed in response to stress and replaced with new bone. A period of rest, or removing weight bearing stresses, is critical to allow new bone formation. If repetitive stress is constantly placed on bone, microfractures may occur as bone formation lags behind resorbtion. Increasing endurance training can further damage bone and propagate stress fractures. Tibial shaft fractures pose a risk of fracture displacement if continued sport participation is allowed. Resolution of symptoms often takes 8-12 months of rest. Estrogen and testosterone are two hormones that influence bone remodeling. Estrogen slows bone resorbtion and helps maintain adequate bone density. Female athletes that participate in endurance sports like long distance running are at risk of low estrogen levels and amenorrhea. Menstrual cycle changes can be influenced by exercise as running 10 miles per week has a 6% incidence of amenorrhea while running 80 miles a week has a 43% incidence of amenorrhea. The patient has a normal diet, normal caloric intake, and a normal bone density which most likely rules out low caloric intake, estrogen levels, and metabolic bone disease as causes of the patient’s stress fracture. 1.2

    Answer B.

     

    References

    1. Tanaka MJ, Szymanski LM, Dale JL, Dixit S, Jones L. Team Approach: Treatment of Injuries in the Female Athlete. JBJS Reviews – Team Approach Review Articles: 22 January 2019 – Volume 7 – Issue 1 – p. e7-e
    2. Shindle, Michael K. MD; Endo, Yoshimi MD; Warren, Russell F. MD; Lane, Joseph M. MD; Helfet, David L. MD; Schwartz, Elliott N. MD; Ellis, Scott J. MD Stress Fractures About the Tibia, Foot, and Ankle, Journal of the American Academy of Orthopaedic Surgeons: March 2012 – Volume 20 – Issue 3 – p 167-176 doi: 10.5435/JAAOS-20-03-167.
    Incorrect

    Excessive endurance training in athletics can alter hormone balances and have negative consequences on the structural integrity of bone. Skeletal bone is constantly remodeled and repaired in response to stress placed during weight bearing activities. Bone is resorbed in response to stress and replaced with new bone. A period of rest, or removing weight bearing stresses, is critical to allow new bone formation. If repetitive stress is constantly placed on bone, microfractures may occur as bone formation lags behind resorbtion. Increasing endurance training can further damage bone and propagate stress fractures. Tibial shaft fractures pose a risk of fracture displacement if continued sport participation is allowed. Resolution of symptoms often takes 8-12 months of rest. Estrogen and testosterone are two hormones that influence bone remodeling. Estrogen slows bone resorbtion and helps maintain adequate bone density. Female athletes that participate in endurance sports like long distance running are at risk of low estrogen levels and amenorrhea. Menstrual cycle changes can be influenced by exercise as running 10 miles per week has a 6% incidence of amenorrhea while running 80 miles a week has a 43% incidence of amenorrhea. The patient has a normal diet, normal caloric intake, and a normal bone density which most likely rules out low caloric intake, estrogen levels, and metabolic bone disease as causes of the patient’s stress fracture. 1.2

    Answer B.

     

    References

    1. Tanaka MJ, Szymanski LM, Dale JL, Dixit S, Jones L. Team Approach: Treatment of Injuries in the Female Athlete. JBJS Reviews – Team Approach Review Articles: 22 January 2019 – Volume 7 – Issue 1 – p. e7-e
    2. Shindle, Michael K. MD; Endo, Yoshimi MD; Warren, Russell F. MD; Lane, Joseph M. MD; Helfet, David L. MD; Schwartz, Elliott N. MD; Ellis, Scott J. MD Stress Fractures About the Tibia, Foot, and Ankle, Journal of the American Academy of Orthopaedic Surgeons: March 2012 – Volume 20 – Issue 3 – p 167-176 doi: 10.5435/JAAOS-20-03-167.
  42. Question 42 of 48
    42. Question

    A 4 year old girl is brought to your office by her parents with right ankle pain after tripping at the playground 2 days ago. She is having difficulty bearing weight on the ankle and she is asking for her parents to carry her to get around. On physical exam she has swelling and tenderness to palpation over the lateral ankle. It is difficult to pinpoint the exact location of the tenderness as she is both tender over the distal fibula and the anterior talofibular ligament. AP and lateral x-rays of the right ankle are shown in figures 1 and 2. Which is true about pediatric ankle injuries?

    Correct

    Ankle injuries are very common in the pediatric population and can be a challenge to determine the cause of pain. Differentiating a Salter Harris (SH) I fracture of the distal fibula vs. a lateral ankle sprain is the most common diagnostic dilemma as both have a similar injury mechanism, physical exam, and radiographic findings. Boutis et al. performed an MRI one week after injury in pediatric patients (mean age of 8) who were diagnosed with SH I fractures and found that over 80% had a ligament injury. A repeat ankle x-ray taken 4 weeks after injury showing periosteal new bone formation is one of the few ways to confirm a SH I fracture.  An ankle sprain and a SH I fracture can both be treated with a walking boot, an air stirrup brace, or a cast for 3-4 weeks with activity modification for 1-2 weeks thereafter. An air stirrup ankle brace offered an earlier return to activities and was preferred over casting by most kids with stable ankle fractures. 1,2

    Answer B.

    References

     

    1. Shirley, Eric D. MD; Maguire, Kathleen Joan MD; Mantica, Abigail Louise MD; Kruse, Richard Wayne DO, MBA. Alternatives to Traditional Cast Immobilization in Pediatric Patients. Journal of the American Academy of Orthopaedic Surgeons 28(1):p e20-e27, January 1, 2020. | DOI: 10.5435/JAAOS-D-18-00152
    2. Hofsli M, Torfing T, Al-Aubaidi Z. The proportion of distal fibula Salter–Harris type I epiphyseal fracture in the paediatric population with acute ankle injury: a prospective MRI study. Journal of Pediatric Orthopaedics B. 2016 Mar 1;25(2):126-32.
    Incorrect

    Ankle injuries are very common in the pediatric population and can be a challenge to determine the cause of pain. Differentiating a Salter Harris (SH) I fracture of the distal fibula vs. a lateral ankle sprain is the most common diagnostic dilemma as both have a similar injury mechanism, physical exam, and radiographic findings. Boutis et al. performed an MRI one week after injury in pediatric patients (mean age of 8) who were diagnosed with SH I fractures and found that over 80% had a ligament injury. A repeat ankle x-ray taken 4 weeks after injury showing periosteal new bone formation is one of the few ways to confirm a SH I fracture.  An ankle sprain and a SH I fracture can both be treated with a walking boot, an air stirrup brace, or a cast for 3-4 weeks with activity modification for 1-2 weeks thereafter. An air stirrup ankle brace offered an earlier return to activities and was preferred over casting by most kids with stable ankle fractures. 1,2

    Answer B.

    References

     

    1. Shirley, Eric D. MD; Maguire, Kathleen Joan MD; Mantica, Abigail Louise MD; Kruse, Richard Wayne DO, MBA. Alternatives to Traditional Cast Immobilization in Pediatric Patients. Journal of the American Academy of Orthopaedic Surgeons 28(1):p e20-e27, January 1, 2020. | DOI: 10.5435/JAAOS-D-18-00152
    2. Hofsli M, Torfing T, Al-Aubaidi Z. The proportion of distal fibula Salter–Harris type I epiphyseal fracture in the paediatric population with acute ankle injury: a prospective MRI study. Journal of Pediatric Orthopaedics B. 2016 Mar 1;25(2):126-32.
  43. Question 43 of 48
    43. Question

    A 16 year old male presents to your office with chronic back pain over the last 2 years. He believes the pain started during baseball season and has continued intermittently during basketball in the winter. He recently had lower back pain while playing soccer where he had a hard time bending his back. He denies any radiation of pain or changes with bowel and bladder function. On physical exam he has tenderness over the L3 spinous process and paraspinous musculature.  He notes to have very tight hamstrings and has trouble bending at the waist and touching his knees. Sagittal MRI of the lumbar spine shows bone edema at the L3 pedicles bilaterally with early spondylolysis at the right L3 (figure 1). There is no evidence of spondylolisthesis. What is the best treatment option?

    Correct

    Spondylolysis is a defect, or stress fracture, of the pars articularis which is part of the posterior vertebral arch. Spondylolysis is a common sports related injury with one study finding a 48% incidence in young athletes with back pain. All of these athletes had negative x-rays and the spondylolysis was found on MRI.  A common presentation includes a history of activity related chronic back pain without radiculopathy and hamstring tightness of physical exam. High risk sports include football, gymnastics, and diving where there is a lot of repetitive lumbar extension and rotation. Standard AP and lateral radiographs should be taken initially but are often negative. Treatment involves bracing with a lumbar corset brace (to restrict extension and rotation) until the patient achieves painless flexion and extension of the lumbar spine. Physical therapy is then initiated with a gradual increase back into activities. Physical therapy focuses on hamstring stretching, core strengthening, and avoidance of lumbar hyperextension. In early spondylolysis, signal intensity on MRI can take an average of 4-6 months to resolve with conservative treatment. The goal of early spondylolysis is to prevent fracture progression, spinal instability, and spondylolisthesis. 1,2

    Answer D.

     

    References

    1. Cavalier, Ralph MD; Herman, Martin J. MD; Cheung, Emilie V. MD; Pizzutillo, Peter D. MD. Spondylolysis and Spondylolisthesis in Children and Adolescents: I. Diagnosis, Natural History, and Nonsurgical Management. Journal of the American Academy of Orthopaedic Surgeons 14(7):p 417-424, July 2006.
    2. Kalichman L, Kim DH, Li L, Guermazi A, Berkin V, Hunter DJ. Spondylolysis and spondylolisthesis: prevalence and association with low back pain in the adult community-based population. Spine (Phila Pa 1976). 2009 Jan 15;34(2):199-205. doi: 10.1097/BRS.0b013e31818edcfd. PMID: 19139672; PMCID: PMC3793342.
    Incorrect

    Spondylolysis is a defect, or stress fracture, of the pars articularis which is part of the posterior vertebral arch. Spondylolysis is a common sports related injury with one study finding a 48% incidence in young athletes with back pain. All of these athletes had negative x-rays and the spondylolysis was found on MRI.  A common presentation includes a history of activity related chronic back pain without radiculopathy and hamstring tightness of physical exam. High risk sports include football, gymnastics, and diving where there is a lot of repetitive lumbar extension and rotation. Standard AP and lateral radiographs should be taken initially but are often negative. Treatment involves bracing with a lumbar corset brace (to restrict extension and rotation) until the patient achieves painless flexion and extension of the lumbar spine. Physical therapy is then initiated with a gradual increase back into activities. Physical therapy focuses on hamstring stretching, core strengthening, and avoidance of lumbar hyperextension. In early spondylolysis, signal intensity on MRI can take an average of 4-6 months to resolve with conservative treatment. The goal of early spondylolysis is to prevent fracture progression, spinal instability, and spondylolisthesis. 1,2

    Answer D.

     

    References

    1. Cavalier, Ralph MD; Herman, Martin J. MD; Cheung, Emilie V. MD; Pizzutillo, Peter D. MD. Spondylolysis and Spondylolisthesis in Children and Adolescents: I. Diagnosis, Natural History, and Nonsurgical Management. Journal of the American Academy of Orthopaedic Surgeons 14(7):p 417-424, July 2006.
    2. Kalichman L, Kim DH, Li L, Guermazi A, Berkin V, Hunter DJ. Spondylolysis and spondylolisthesis: prevalence and association with low back pain in the adult community-based population. Spine (Phila Pa 1976). 2009 Jan 15;34(2):199-205. doi: 10.1097/BRS.0b013e31818edcfd. PMID: 19139672; PMCID: PMC3793342.
  44. Question 44 of 48
    44. Question

    A 9 year old girl presents to the ED with right sided leg pain after a bicycle accident a few hours earlier. She was thrown from her bike and landed on her right side. She was unable to bear weight and in significant pain when her parents brought her to the ED. AP x-ray of the right femur is shown in figure 1. On physical exam she has a slightly shorted and rotated right lower extremity. Her foot is pink and warm and all distal pulses are intact. What is the best treatment option for this patient?

    Correct

    Femur fractures in the pediatric population are relatively uncommon. Femur fractures that occur in children under 3 years of age should be worked up for potential child abuse. Treatment of these fractures is very much age dependent with ages 0-6 months generally treated with a Pavlik harness, ages 6 months to 4-5 years with a spica cast, and 4 to 11 years with surgery. Children 5 and under generally heal quickly and have a thick periosteum which provides additional fracture stability. Surgical fixation of patients 4-5 years and older generally includes either flexible intramedullary nails, submuscular bridge plate fixation, or less commonly, external fixation. Intramedullary nails are ideal for transverse midshaft fractures without distal or proximal extension. Patients with length unstable fractures and proximal or distal extension are good candidates for submuscular bridge plate fixation. The plate is placed at least 2 cm from the physis when possible to avoid growth disturbance and is generally removed when full healing is present. 1,2

    Answer D.

     

    References

    1. Greenhill, Dustin A. MD; Herman, Martin J. MD, FAAOS. Treatment of Pediatric Femoral Shaft Fractures. Journal of the American Academy of Orthopaedic Surgeons 30(22):p e1443-e1452, November 15, 2022. | DOI: 10.5435/JAAOS-D-22-00415
      2. Li, Ying MD; Hedequist, Daniel J. MD. Submuscular Plating of Pediatric Femur Fracture. Journal of the American Academy of Orthopaedic Surgeons 20(9):p 596-603, September 2012. | DOI: 10.5435/JAAOS-20-09-596
    Incorrect

    Femur fractures in the pediatric population are relatively uncommon. Femur fractures that occur in children under 3 years of age should be worked up for potential child abuse. Treatment of these fractures is very much age dependent with ages 0-6 months generally treated with a Pavlik harness, ages 6 months to 4-5 years with a spica cast, and 4 to 11 years with surgery. Children 5 and under generally heal quickly and have a thick periosteum which provides additional fracture stability. Surgical fixation of patients 4-5 years and older generally includes either flexible intramedullary nails, submuscular bridge plate fixation, or less commonly, external fixation. Intramedullary nails are ideal for transverse midshaft fractures without distal or proximal extension. Patients with length unstable fractures and proximal or distal extension are good candidates for submuscular bridge plate fixation. The plate is placed at least 2 cm from the physis when possible to avoid growth disturbance and is generally removed when full healing is present. 1,2

    Answer D.

     

    References

    1. Greenhill, Dustin A. MD; Herman, Martin J. MD, FAAOS. Treatment of Pediatric Femoral Shaft Fractures. Journal of the American Academy of Orthopaedic Surgeons 30(22):p e1443-e1452, November 15, 2022. | DOI: 10.5435/JAAOS-D-22-00415
      2. Li, Ying MD; Hedequist, Daniel J. MD. Submuscular Plating of Pediatric Femur Fracture. Journal of the American Academy of Orthopaedic Surgeons 20(9):p 596-603, September 2012. | DOI: 10.5435/JAAOS-20-09-596
  45. Question 45 of 48
    45. Question

    A 13 year old female presents to your office with right hip pain while running. She started track practices a few weeks ago and has noticed pain and a snapping feeling in her right hip after a few laps around the track. She denies a previous injury or precipitating event, although she says her aunt has hip instability from hip dysplasia. AP and lateral x-rays of the right hip are shown in figures 1 and 2. On AP x-ray of the patients right hip the lateral center-edge angle measures 30 degrees. On physical exam the patient has a negative FABER test and pain with resisted hip flexion. What is the next best step in treatment for this patient?

    Correct

    The differential diagnosis for hip pain in an athletic adolescent can be broad. Coxa saltans, otherwise known as “snapping hip syndrome”, can include a tight IT band snapping over the greater trochanter and catching of the iopsoas tendon at the iliopectineal eminence or on the femoral head. Other common diagnoses may include tight hip flexors, hip dysplasia, acetabular labral tear, and femoroacetabular impingement. Hip dysplasia generally produces symptoms of hip instability (popping sensation, pain with activities) and is associated with chronic symptoms for years. Radiographs of the hip will show decreased femoral head coverage with a lateral center-edge angle (LCEA) <20 degrees. Femoroacetabular impingement (FAI) causes pain with hip flexion and rotation and patients may describe their pain using a C sign by cupping the anterior lateral aspect of the hip with the thumb and forefingers. FAI is opposite to hip dysplasia as the condition is caused by over coverage of the femoral head by the acetabulum. FAI usually has an increased LCEA >40 degrees. Conservative treatment is initially recommended for all adolescent patients presenting with athletic related hip pain. A period of rest and NSAIDS can usually settle down an inflammatory condition like tendinitis or hip joint synovitis. Physical therapy is also important to stretch tight muscles and tendons and to strengthen the hip joint for athletic performance. 1,2

    Answer A.

     

    References

    1. Schmitz, Matthew R. MD; Murtha, Andrew S. MD;  The ANCHOR Study Group; Clohisy, John C. MD. Developmental Dysplasia of the Hip in Adolescents and Young Adults. Journal of the American Academy of Orthopaedic Surgeons 28(3):p 91-101, February 1, 2020.
    2. Allen, William C. MD; Cope, Ray MD. Coxa Saltans: The Snapping Hip Revisited. Journal of the American Academy of Orthopaedic Surgeons 3(5):p 303-308, September 1995.

     

    Incorrect

    The differential diagnosis for hip pain in an athletic adolescent can be broad. Coxa saltans, otherwise known as “snapping hip syndrome”, can include a tight IT band snapping over the greater trochanter and catching of the iopsoas tendon at the iliopectineal eminence or on the femoral head. Other common diagnoses may include tight hip flexors, hip dysplasia, acetabular labral tear, and femoroacetabular impingement. Hip dysplasia generally produces symptoms of hip instability (popping sensation, pain with activities) and is associated with chronic symptoms for years. Radiographs of the hip will show decreased femoral head coverage with a lateral center-edge angle (LCEA) <20 degrees. Femoroacetabular impingement (FAI) causes pain with hip flexion and rotation and patients may describe their pain using a C sign by cupping the anterior lateral aspect of the hip with the thumb and forefingers. FAI is opposite to hip dysplasia as the condition is caused by over coverage of the femoral head by the acetabulum. FAI usually has an increased LCEA >40 degrees. Conservative treatment is initially recommended for all adolescent patients presenting with athletic related hip pain. A period of rest and NSAIDS can usually settle down an inflammatory condition like tendinitis or hip joint synovitis. Physical therapy is also important to stretch tight muscles and tendons and to strengthen the hip joint for athletic performance. 1,2

    Answer A.

     

    References

    1. Schmitz, Matthew R. MD; Murtha, Andrew S. MD;  The ANCHOR Study Group; Clohisy, John C. MD. Developmental Dysplasia of the Hip in Adolescents and Young Adults. Journal of the American Academy of Orthopaedic Surgeons 28(3):p 91-101, February 1, 2020.
    2. Allen, William C. MD; Cope, Ray MD. Coxa Saltans: The Snapping Hip Revisited. Journal of the American Academy of Orthopaedic Surgeons 3(5):p 303-308, September 1995.

     

  46. Question 46 of 48
    46. Question

    A 13 year old male presents to your office with right hip pain after a fall during a soccer match earlier in the day. He admits to a sudden stop and falling forward with pain. He is now having difficulty bearing weight on the right leg and cannot flex his hip. AP pelvis x-ray is shown in figure 1. What is the next best step in treatment of this patient?

    Correct

    The patient has an avulsion fracture of the less trochanter. Avulsion fractures are commonly seen in the pediatric population at secondary ossification centers including the ischial tuberosity, anterior superior iliac spine, and anterior inferior iliac spine, and less often in the tibial tubercle, calcaneus, and greater and lesser trochanters. Secondary ossification centers act as attachment sites for tendons and ligaments and don’t contribute to the length of bone. Secondary ossification centers generally fuse by 17 years of age so the most common age group for avulsion fractures are adolescent athletes. In adolescents the tendon attachment is stronger than the growth plate so sudden contraction of the attached tendon/muscle can result in an avulsion injury. Avulsion injuries are diagnosed radiographically in most cases and advanced imaging is not necessary unless the diagnosis is in question. Isolated lesser trochanter avulsion fractures occur with a sudden contraction of the iliopsoas muscle (hip flexor). Patients present with groin pain, difficulty ambulating, and inability to flex the hip. Lesser trochanter avulsion fractures are treated non-operatively with a 3-4 week period of rest and crutches for support, followed by a gradual rehab back into athletic activities. Sports should be resumed when the patient is asymptomatic and there are radiographic signs of healing, which may take 2-3 months. The prognosis for lesser trochanter avulsion injuries is excellent and surgery is only considered if patients develop a symptomatic nonunion and/or a painful exostosis. Surgery involves removal of the painful nonunion with reattachment of the iliopsoas to the proximal femur. 1,2

    Answer C.

     

    References

    1. Tahir T, Manzoor QW, Gul IA, Bhat SA, Kangoo KA. Isolated Avulsion Fractures of Lesser Trochanter in Adolescents – A Case Series and Brief Literature Review. J Orthop Case Rep. 2019 Jan-Feb;9(1):11-14.
    2. Schiller, Jonathan MD; DeFroda, Steven MD, ME; Blood, Travis MD. Lower Extremity Avulsion Fractures in the Pediatric and Adolescent Athlete. Journal of the American Academy of Orthopaedic Surgeons 25(4):p 251-259, April 2017.
    Incorrect

    The patient has an avulsion fracture of the less trochanter. Avulsion fractures are commonly seen in the pediatric population at secondary ossification centers including the ischial tuberosity, anterior superior iliac spine, and anterior inferior iliac spine, and less often in the tibial tubercle, calcaneus, and greater and lesser trochanters. Secondary ossification centers act as attachment sites for tendons and ligaments and don’t contribute to the length of bone. Secondary ossification centers generally fuse by 17 years of age so the most common age group for avulsion fractures are adolescent athletes. In adolescents the tendon attachment is stronger than the growth plate so sudden contraction of the attached tendon/muscle can result in an avulsion injury. Avulsion injuries are diagnosed radiographically in most cases and advanced imaging is not necessary unless the diagnosis is in question. Isolated lesser trochanter avulsion fractures occur with a sudden contraction of the iliopsoas muscle (hip flexor). Patients present with groin pain, difficulty ambulating, and inability to flex the hip. Lesser trochanter avulsion fractures are treated non-operatively with a 3-4 week period of rest and crutches for support, followed by a gradual rehab back into athletic activities. Sports should be resumed when the patient is asymptomatic and there are radiographic signs of healing, which may take 2-3 months. The prognosis for lesser trochanter avulsion injuries is excellent and surgery is only considered if patients develop a symptomatic nonunion and/or a painful exostosis. Surgery involves removal of the painful nonunion with reattachment of the iliopsoas to the proximal femur. 1,2

    Answer C.

     

    References

    1. Tahir T, Manzoor QW, Gul IA, Bhat SA, Kangoo KA. Isolated Avulsion Fractures of Lesser Trochanter in Adolescents – A Case Series and Brief Literature Review. J Orthop Case Rep. 2019 Jan-Feb;9(1):11-14.
    2. Schiller, Jonathan MD; DeFroda, Steven MD, ME; Blood, Travis MD. Lower Extremity Avulsion Fractures in the Pediatric and Adolescent Athlete. Journal of the American Academy of Orthopaedic Surgeons 25(4):p 251-259, April 2017.
  47. Question 47 of 48
    47. Question


    A 16 year old female presents to your office with right elbow pain after a fall 2 days ago. She was walking to the bus when she tripped on ice and landed on the elbow. The pain has improved significantly but her mom wanted her to be checked out. She denies a history of numbness or cold insensitivity to the hand and forearm. On physical exam she has no swelling or bruising to the elbow and slight tenderness to palpation over the olecranon. X-rays of the elbow show no evidence of a fracture, however she has a hook-shaped bony mass on the supracondylar aspect of her proximal humerus shown in Figure 1. What x-ray feature of this supracondylar process helps distinguish the bony mass from an osteochondroma?

    Correct

    Benign bone growth that extends outward from normal bone (exostosis) are often seen incidentally on x-ray. The most common exophytic lesion diagnosed incidentally on x-ray is an osteochondroma. The classic findings of an osteochondroma include a pedunculated bony lesion in the metaphyseal region that projects away from the nearby joint. A supracondylar process (or spur) on the anteromedial aspect of the humerus (about 5 cm proximal to the medial epicondyle) has similar radiographic findings and can be confused with an osteochondroma. A fibrous band of tissue known as the ligament of Struthers attaches to the process and can cause symptomatic compression of the of the median nerve and/or brachial artery. The lesion should be differentiated from an osteochondroma. The patient’s images show the exostosis pointing toward the joint whereas an osteochondroma generally points away. The humeral cortex is continuous with an osteochondroma cortex whereas the humeral cortex is intact with a supracondylar process. 1,2
    Answer B.
    References
    1. Mutnuru PC, Perubhotla LM. Rare Mimickers of Exostosis: A Case Series. J Clin Diagn Res. 2016 Jul;10(7):TR06-7. doi: 10.7860/JCDR/2016/18794.8193. Epub 2016 Jul 1. PMID: 27630926; PMCID: PMC5020238.
    2. Shivaleela C, Suresh BS, Kumar GV, Lakshmiprabha S. Morphological study of the supracondylar process of the humerus and its clinical implications. Journal of clinical and diagnostic research: JCDR. 2014 Jan;8(1):1.

    Incorrect

    Benign bone growth that extends outward from normal bone (exostosis) are often seen incidentally on x-ray. The most common exophytic lesion diagnosed incidentally on x-ray is an osteochondroma. The classic findings of an osteochondroma include a pedunculated bony lesion in the metaphyseal region that projects away from the nearby joint. A supracondylar process (or spur) on the anteromedial aspect of the humerus (about 5 cm proximal to the medial epicondyle) has similar radiographic findings and can be confused with an osteochondroma. A fibrous band of tissue known as the ligament of Struthers attaches to the process and can cause symptomatic compression of the of the median nerve and/or brachial artery. The lesion should be differentiated from an osteochondroma. The patient’s images show the exostosis pointing toward the joint whereas an osteochondroma generally points away. The humeral cortex is continuous with an osteochondroma cortex whereas the humeral cortex is intact with a supracondylar process. 1,2
    Answer B.
    References
    1. Mutnuru PC, Perubhotla LM. Rare Mimickers of Exostosis: A Case Series. J Clin Diagn Res. 2016 Jul;10(7):TR06-7. doi: 10.7860/JCDR/2016/18794.8193. Epub 2016 Jul 1. PMID: 27630926; PMCID: PMC5020238.
    2. Shivaleela C, Suresh BS, Kumar GV, Lakshmiprabha S. Morphological study of the supracondylar process of the humerus and its clinical implications. Journal of clinical and diagnostic research: JCDR. 2014 Jan;8(1):1.

  48. Question 48 of 48
    48. Question

    A 5 year old boy is brought to your office with his mother with complaints of right hip pain for 2 days. He has been limping and doesn’t want to bear weight on the right side. The child points to the right hip where the pain is coming from. His mom denies any current fevers or chills or cold symptoms before the hip pain started. On AP and lateral x-rays of the right hip there is irregularity of the femoral head concerning of avascular necrosis/Legg-Calvé-Perthes disease (figures 1 and 2). The patient is afebrile on exam and he walks with a noticeable limp favoring the right side. He has moderate pain with hip flexion and rotation. What is the next best step in treatment?

    Correct

    Legg-Calvé-Perthes disease is caused by a loss of blood flow to the femoral head in a skeletally immature patient with 4-8 years of age being the most common ages of presentation. The condition typically follows a predictable radiographic course of bone necrosis, bone resorption, and reossification.  Reossification is typically evident 6 months from onset of symptoms. Radiographs are the preferred imaging modality to diagnose, stage, a follow Perthes. Serial radiographs follow the sphericity of the femoral head and stability of the hip joint. Collapse of the lateral femoral head represents an unstable pattern that can cause hip instability. Age is also a predictor of outcome with the diagnosis made before the age of 6 years of age offering a better prognosis. Serial radiographs should be taken every 3-4 months until reossification to follow femoral head changes, with the ultimate goal of preventing premature hip arthritis. Patients who present <6 years of age can be treated with observation with weight bearing as tolerated and activity restrictions if the hip is symptomatic. Patients who present at age >6-8 years of age with significant femoral head collapse may require bracing, nonweight bearing and/or surgical fixation. 1,2

    Answer A.

    References

    1. Laine JC, Martin BD, Novotny SA, Kelly DM. Role of advanced imaging in the diagnosis and management of active Legg-Calve-Perthes disease. JAAOS-Journal of the American Academy of Orthopaedic Surgeons. 2018 Aug 1;26(15):526-36.
    2. Kim, Harry K. W. MD. Legg-Calvé-Perthes Disease. American Academy of Orthopaedic Surgeon 18(11):p 676-686, November 2010.
    Incorrect

    Legg-Calvé-Perthes disease is caused by a loss of blood flow to the femoral head in a skeletally immature patient with 4-8 years of age being the most common ages of presentation. The condition typically follows a predictable radiographic course of bone necrosis, bone resorption, and reossification.  Reossification is typically evident 6 months from onset of symptoms. Radiographs are the preferred imaging modality to diagnose, stage, a follow Perthes. Serial radiographs follow the sphericity of the femoral head and stability of the hip joint. Collapse of the lateral femoral head represents an unstable pattern that can cause hip instability. Age is also a predictor of outcome with the diagnosis made before the age of 6 years of age offering a better prognosis. Serial radiographs should be taken every 3-4 months until reossification to follow femoral head changes, with the ultimate goal of preventing premature hip arthritis. Patients who present <6 years of age can be treated with observation with weight bearing as tolerated and activity restrictions if the hip is symptomatic. Patients who present at age >6-8 years of age with significant femoral head collapse may require bracing, nonweight bearing and/or surgical fixation. 1,2

    Answer A.

    References

    1. Laine JC, Martin BD, Novotny SA, Kelly DM. Role of advanced imaging in the diagnosis and management of active Legg-Calve-Perthes disease. JAAOS-Journal of the American Academy of Orthopaedic Surgeons. 2018 Aug 1;26(15):526-36.
    2. Kim, Harry K. W. MD. Legg-Calvé-Perthes Disease. American Academy of Orthopaedic Surgeon 18(11):p 676-686, November 2010.
Proudly powered by WordPress | Theme: Motif by WordPress.com.